Torts Flashcards

ALA Quizzes, Kaplan Quizzes, Kaplan Qbank, CLS Quizzes

1
Q

Three friends were driving home together from a local tavern. The driver had not consumed any alcoholic
beverages at the tavern. On their way home, the three friends observed a man wandering haphazardly
across the busy highway. One friend remarked, “I hope that guy makes it home. If he’s not careful, he could get killed.” The driver said, “That’s his problem. I have to get home. I can’t be bothered to stop and help him.” After being dropped off at his home, the friend felt pangs of guilt and decided to go back and help the man that he’d seen. The friend asked his brother, who had not been drinking, to drive back to find and help the man. When the brothers returned to the area where the man had been, they found him lying on the shoulder of the highway, obviously injured. They helped the man into their car and the brother drove the man to the hospital. When they arrived at the hospital, the man was admitted with a broken leg. The leg had been broken when the man was wandering on the highway and was struck by a vehicle which had left the scene. The broken leg was the man’s only injury.

If the man sues the driver of the car the friend had been in on the way home from the tavern for damages for his injuries, will he prevail?

(A) No, because the driver was under no duty to help the man.

(B) No, because the man was involved in an abnormally dangerous activity.

(C) Yes, because a reasonable person would have helped the man.

(D) Yes, because the driver breached his duty to the man.

A

(A) No, because the driver was under no duty to help the man.

How well did you know this?
1
Not at all
2
3
4
5
Perfectly
2
Q

While a man was cutting his front lawn with a rotary power lawn mower, his neighbor was coming home from work. The neighbor was walking past the man’s front lawn when he was hit in the eye with a small rock discharged from the lawn mower. The neighbor lost the use of his eye. The lawn mower had no space for any protective device such as metal netting or a drag plate to prevent the ejection of projectiles such as rocks or pieces of wood.

In a suit for damages based on strict liability against the manufacturer of the mower, will the neighbor prevail?

A. Yes, because he will be able to prove a manufacturing defect.

B. Yes, because the lawn mower was defectively designed.

C. No, because the neighbor is not a proper plaintiff in a strict liability action.

D. No, because the neighbor was negligent in walking on the sidewalk while the lawn mower was being used.

A

C. No, because the neighbor is not a proper plaintiff in a strict liability action.

How well did you know this?
1
Not at all
2
3
4
5
Perfectly
3
Q

A novice rock climber was acquiring all of the equipment he needed to pursue his new favorite hobby. He saw an advertisement for a sale on Steel Strength heavy duty rope at a sporting goods store. The manufacturer’s ad showed a photo of an athletic man standing next to a jagged rock face with a rope tied around his waist in rock-climbing fashion. The ad read: “Steel Strength rope helps me climb where I need to climb and go where I need to go.” The rock climber bought a spool of Steel Strength rope at the store. He didn’t discuss the rope with the sales clerk because, based on the ad, he was confident that it was what he needed. The climber brought the rope on his next climbing excursion. The climber’s large friend, who had also just started climbing, came along, and he borrowed some of the Steel Strength rope. The rope, which really wasn’t designed for supporting even an average-weight person, broke under his weight. He fell from the rock face, landing on a park ranger. The friend broke both of his legs, and the ranger broke her arm. The friend and the park ranger sue the manufacturer in product liability for their injuries.

Which of the following is correct?

A. Neither the ranger nor the friend can prevail, because they were not foreseeable plaintiffs.

B. Neither the ranger nor the friend can prevail against the manufacturer, because the novice climber did not ask the store clerk for help.

C. The ranger cannot prevail against the manufacturer, because she was working at the time of the accident.

D. Both the ranger and the friend can prevail against the manufacturer, because the advertisement created an express warranty.

A

D. Both the ranger and the friend can prevail against the manufacturer, because the advertisement created an express warranty.

How well did you know this?
1
Not at all
2
3
4
5
Perfectly
4
Q

After a power outage, the stoplights at a busy intersection were blinking red for traffic going in every direction. By statute, motorists must come to a full stop at a blinking red traffic signal before proceeding through the intersection. Cars driven by the plaintiff and by the defendant arrived at the intersection at the same time. Due to inattention, neither one stopped for the signal and the cars collided in the intersection.

The plaintiff sued the defendant for his injuries. The trier of fact determined that the plaintiff was more at fault than the defendant.

Will the plaintiff likely recover damages?

A. No, because the plaintiff did not stop at the blinking red light.

B. No, because the plaintiff’s fault was greater than the defendant’s.

C. Yes, if the defendant had the last clear chance to avoid the accident.

D. Yes, even though the plaintiff’s fault was greater than the defendant’s.

A

D. Yes, even though the plaintiff’s fault was greater than the defendant’s.

How well did you know this?
1
Not at all
2
3
4
5
Perfectly
5
Q

On New Year’s Eve, a man negligently hit a woman with his car. The woman was rushed to the hospital and sent into surgery. The surgeon, who was summoned from a nearby party, operated on the woman while
intoxicated. As a result, the surgeon failed to remove a drain tube, causing the woman harm and requiring a second operation to remove. The woman has now brought suit against the man for her injuries.
At trial, testimony showed that the woman suffered $100,000 in damages caused to her legs from the accident and $50,000 from complications surrounding the drain tube. At trial, an expert testified that the surgeon’s efforts fell below the standard of care that should have been exercised in an operating room as a result of his intoxication. The jury also found that the woman was 25% responsible for the accident.

How much should the woman recover from the man?

(A) $75,000, or the amount of her injuries from the accident minus her own negligence.

(C) $125,000, or the woman’s injuries from the accident as well as the doctor’s negligence, minus her own negligence.

(D) $150,000, or the full amount of her injuries for both the accident and the doctor’s negligence.

(B) $100,000, or the full amount of her injuries from the accident.

A

(A) $75,000, or the amount of her injuries from the accident minus her own negligence.

How well did you know this?
1
Not at all
2
3
4
5
Perfectly
6
Q

A family lived in a quiet residential neighborhood. They built a treehouse in the backyard for their three adolescent children. One weekend, the family took a vacation to a neighboring state. While the family was away, a 10-year-old boy who lived in the neighborhood trespassed upon the property. The boy was in the family’s backyard when he saw the treehouse, which was situated in a tall tree. The boy climbed up the ladder and began playing in the treehouse. While inside, the boy peered out the window and saw a large limb with avocados perched above the treehouse.

The boy decided to collect a few avocados to bring home to his mother, but found that he was unable to reach them. Despite knowing it was dangerous, he climbed on top of the treehouse roof in an effort to get closer to the branch. As the boy was walking along the roof, he slipped and fell to the ground, severely fracturing his ankle and leg.

If the boy’s parents assert a tort action against the family to recover for the boy’s injuries, will the plaintiffs prevail?

A. Yes, because the treehouse constituted an attractive nuisance.

B. Yes, because the boy’s own negligence was the cause of his injury.

C. No, because a child of like age, intelligence, and experience would not have climbed on top of the treehouse roof.

D. No, because the boy recognized the risk that was involved.

A

D. No, because the boy recognized the risk that was involved.

Discussion of correct answer: This is an extremely tricky Multistate question dealing with the attractive nuisance doctrine.

Under the attractive nuisance doctrine, a possessor of land is subject to liability for physical harm to trespassing children where:

(1) an artificial dangerous condition exists on the property;
(2) the possessor knows or should know that children are likely to trespass on the property;
(3) the children because of their youth, age, or immaturity fail to appreciate the danger or realize the risks involved in intermeddling with it; and
(4) the utility of maintaining the dangerous condition is slight compared to the risks involved.

It is important to note that the inability of a child to appreciate the danger is a subjective test. In general, for this subjective test, the child, because of his lack of age and maturity, either must not appreciate the danger involved or must not discover the condition. Because the facts state that the boy knew that climbing out onto the treehouse roof was dangerous but did so anyway, the plaintiffs will not be liable for the boy’s injuries.

How well did you know this?
1
Not at all
2
3
4
5
Perfectly
7
Q

A motorist hits a pedestrian and leaves the pedestrian laying on the side of the road, in need of medical attention. As a result of the accident, the pedestrian suffered broken ribs and a concussion. Shortly after the motorist drove away, a passerby found the pedestrian and robbed him. In the course of the robbery, the passerby broke the pedestrian’s arm.

Will the motorist be held liable for the pedestrian’s broken arm?

A. Yes, because the passerby’s action was a superseding cause.

B. Yes, because the passerby’s action was an intervening cause.

C. No, because the passerby’s action was a superseding cause.

D. No, because the passerby’s action was an intervening cause.

A

C. No, because the passerby’s action was a superseding cause.

Discussion of correct answer: A superseding cause is an unforeseeable, intervening cause that breaks the chain of causation between the initial wrongful act and the ultimate injury, and thus relieves the original tortfeasor of any further liability. An example of a superseding cause is an intentional tort of a third person. On the other hand, an intervening force is one that actively operates in producing harm to another after the actor has already committed his negligent act or omission. As a general rule, a defendant will be held liable for harm caused by foreseeable intervening forces. Here, the passerby committed an intentional tort by robbing the victim. As such, the passerby’s broken arm was caused by a superseding cause and the motorist will not be liable for this particular injury.

How well did you know this?
1
Not at all
2
3
4
5
Perfectly
8
Q

A driver wanted to purchase a new SUV but could not afford the retail prices being offered by the new car dealers in the area. While reading the newspaper, the driver saw an advertisement by an auction company. In the advertisement, the auction company claimed that it was selling new, unused, late-model cars for substantially lower prices than those charged by automobile dealers representing manufacturers. The driver went to the automobile auction and placed the winning bid on a late-model SUV. As he was driving the vehicle home at a high rate of speed, the brakes suddenly failed, causing an accident. The brake failure resulted from a defectively designed brake mechanism. The driver suffered a broken leg and brought suit against the auction company in strict liability to recover for his injuries.

What must the driver prove in order to be successful in his lawsuit?

A. The auction company failed to inspect the brakes and a cursory inspection would have disclosed the defect.

B. The auction company was in the business of selling automobiles.

C. The driver’s excessive speed was not a legal cause of the accident.

D. The driver inspected the brakes himself before driving home from the auction.

A

B. The auction company was in the business of selling automobiles.

How well did you know this?
1
Not at all
2
3
4
5
Perfectly
9
Q

A homeowner in a small city-owned property that was bordered by a local roadway. On the very edge of the property, next to the homeowner’s fence, was a large oak tree that hung over the sidewalk beside the roadway. One day a large limb that was decayed broke off and injured a passerby as he was walking on the sidewalk to the grocery store. It was conceded by the homeowner that he was unable to properly maintain the tree due to a recent illness.

If the passerby subsequently brings an action against the homeowner, will he recover?

A. Yes, because the homeowner owed a reasonable duty of care to persons outside the premises.

B. Yes, because the homeowner owed a nondelegable duty to persons outside the premises.

C. No, because the passerby assumed the risk by walking under the tree limb.

D. No, because the homeowner acted reasonably considering his physical condition.

A

A. Yes, because the homeowner owed a reasonable duty of care to persons outside the premises.

How well did you know this?
1
Not at all
2
3
4
5
Perfectly
10
Q

When a hurricane suddenly increased its intensity and changed its path to strike the oceanfront town that a tourist was visiting, she took refuge at the hotel where she was a guest. As the winds increased and the storm surge began to cause the water to rise, the hotel manager advised everyone still in the building to move off of the ground floor. However, the tourist panicked at being trapped by the water and decided to flee the hotel and seek higher ground. She got almost to the main gate of the hotel when she was swept away by the floodwaters and drowned.

In an action by the tourist’s estate against the hotel, what is the likely result?

A. The tourist’s estate will prevail because the tourist was a guest at the hotel.

B. The tourist’s estate will prevail because the hotel manager did not prevent the tourist from leaving.

C. The hotel will prevail because it did not breach any duty it owed to the tourist.

D. The hotel will prevail because the hurricane was an unforeseeable act of God.

A

C. The hotel will prevail because it did not breach any duty it owed to the tourist.

How well did you know this?
1
Not at all
2
3
4
5
Perfectly
11
Q

A popular electronics store is having a Black Friday super sale, with deep price reductions and giveaways for the first five people to make purchases. A man decided to enter the store the night before, hide out in one of the large appliance boxes that were lying in the stockroom, and then be the first person in the store the next morning.

During the night, the man needed to use the restroom, and entered a door marked “Employees Only.” The door led to a stairway that had a broken first step, and the man fell, injuring himself. The next morning, he was found with a broken leg, and arrested.

If the man brings suit against the electronics store, what is the likely result?

A. The man will recover, because he was an invitee who was injured on the premises.

B. The man will recover, because he was a licensee who was injured on the premises.

C. The man will not recover, because he was a trespasser on the premises.

D. The man will not recover, because he was a trespasser in an area marked “Employees Only.”

A

C. The man will not recover, because he was a trespasser on the premises.

How well did you know this?
1
Not at all
2
3
4
5
Perfectly
12
Q

A teenager was traveling northbound through an intersection with a green light. At the moment she entered the intersection, two drivers, one traveling eastbound and one traveling westbound, also entered the intersection. Both of these drivers ran red lights as they entered the intersection, and hit the car driven by the teenager at the same moment. As a result of the accident, the teenager suffered a number of serious injuries.

In a negligence suit by the teenager against the two drivers, what is the likely result?

A. Neither driver will be liable, because the court will apply the but-for test.

B. Both drivers will be liable under the but-for test.

C. Neither driver will be liable, because the court will apply the substantial factor test.

D. Both drivers will be liable under the substantial factor test.

A

D. Both drivers will be liable under the substantial factor test.

Discussion of correct answer: The court may apply the but-for test in a negligence action, and find that a defendant is liable for the plaintiff’s injuries if the defendant’s conduct was the cause-in-fact of an event, and if that event would not have occurred but-for the existence of the conduct. However, where the conduct of multiple defendants results in injury to the plaintiff, the court will not use this test. This is because both defendants can deny liability by pointing to the conduct of the other defendant. Therefore, the court may apply the substantial factor test in such situations. Under this test, a defendant’s conduct is also the cause-in-fact of a plaintiff’s injury if that conduct was a substantial factor in bringing about the injury. Here, the conduct of each driver was a substantial factor in bringing about the teenager’s injuries. As such, the court will find that both drivers are liable under the substantial factor test.

How well did you know this?
1
Not at all
2
3
4
5
Perfectly
13
Q

A tall building was under construction in the downtown business district of a city. A lawyer took a car to the downtown area to go to a meeting with a client. She was running late for the meeting and could not find a legal parking spot, so she decided to park illegally in front of a fire hydrant. This was in violation of a local ordinance that prohibited parking within 50 feet of a fire hydrant. The lawyer figured that it was better to get a ticket than to miss her meeting with an important client. The spot where she parked was next to the site of the construction of the new building. While the lawyer was at her meeting, an accident occurred at the construction site. A large crane was being used to lift a load of bricks. A cable on the crane broke, and the bricks fell. Most of them landed on top of the lawyer’s car. The load of bricks that fell on the lawyer’s car caused the gas tank of the car to rupture and explode. Shrapnel from the explosion flew in all directions and injured a pedestrian who was walking on the sidewalk near the lawyer’s car.

If the pedestrian sues the lawyer and relies on the doctrine of negligence per se, which of the following, if true, is the lawyer’s best defense?

A. Payment of a small fine is the only penalty provided in the ordinance for those who park too close to fire hydrants.

B. The police never issued a ticket to the lawyer for parking in front of the fire hydrant.

C. The purpose of the parking ordinance was to facilitate access to the hydrant by fire trucks, not to protect against accidents like the one that occurred when bricks fell on the lawyer’s car

D, The pedestrian would not have been injured if the construction company had properly maintained the crane.

A

C. The purpose of the parking ordinance was to facilitate access to the hydrant by fire trucks, not to protect against accidents like the one that occurred when bricks fell on the lawyer’s car

How well did you know this?
1
Not at all
2
3
4
5
Perfectly
14
Q

A bike messenger was injured when the driver of a car, who was not paying attention to the road, drifted into the bicycle lane and hit him. After the accident, the messenger went to a doctor, who advised that, although he suffered no broken bones, he nevertheless needed to stay off his feet for a few weeks in order to allow his body to recover. However, the messenger had just graduated from college, and needed to start repaying his loans. As a result, he could not discontinue his job as a messenger.

The next week, as the messenger was making a routine delivery in an office building, his legs suddenly gave way, causing him to tumble down several flights of stairs. He was rushed to the hospital, where he was treated for two broken legs and a shattered hip.

If the messenger now sues the driver of the car, which of the following statements is the most accurate?

A. The driver is responsible for all injuries sustained by the messenger, under the eggshell plaintiff rule.

B. The driver is responsible for all injuries sustained by the messenger, because they were foreseeable.

C. The driver is not responsible for all injuries sustained by the messenger, because the doctor warned the messenger to stay off his feet.

D. The driver is not responsible for all injuries sustained by the messenger, under the doctrine of avoidable consequences.

A

D. The driver is not responsible for all injuries sustained by the messenger, under the doctrine of avoidable consequences.

How well did you know this?
1
Not at all
2
3
4
5
Perfectly
15
Q

A law student, who was involved in Law Review, moot court, and clinic, was taking 18 credits. She was afraid she was never going to get everything done on time, and she thought some extra caffeine would help. The law student bought a coffee pot from Coffee Warehouse, Inc. She paid top dollar for the coffee maker, but she figured it was worth it because the coffee pot had a special “refresh” function that would keep brewed coffee fresh over a 48-hour period. The law student used the coffee maker for several weeks and noticed a marked improvement in her energy level. One morning, as the law student was reaching for the coffee pot, the pot suddenly exploded. The law student’s hand was severely cut by flying glass from the shattered pot. An investigation revealed that the glass used by Coffee Warehouse to make the coffee pot was defective, which is why it exploded.

If the law student sues Coffee Warehouse on a strict liability theory, what must she prove to prevail?

A. The coffee pot was defective, and a thorough inspection by Coffee Warehouse’s would have revealed the defect.

B. The coffee pot was defective.

C. The coffee pot was defective, and the defect was the cause of the law student’s injuries.

D. The coffee pot was defective, was thoroughly inspected by Coffee Warehouse, and the defect went undetected.

A

C. The coffee pot was defective, and the defect was the cause of the law student’s injuries.

How well did you know this?
1
Not at all
2
3
4
5
Perfectly
16
Q

A builder specialized in the renovation of old buildings. His most recent project was the conversion of a housing project into luxury condominiums. One of the buildings had been renovated by the city 15 years ago when steel guard rails had been installed on all the windows to prevent children from accidental falls. The builder did most of the exterior painting and brick repair work himself to save money. He greatly improved the appearance of the old buildings by painting the window guards an attractive decorative garden pattern. When renovations were completed, members of the local fire department inspected the premises. The fire inspector was a large, heavy-set man. For public relations and safety purposes, he conducted his inspections in full uniform—big steel-toed black boots, a hard hat, tool belt, keys, and a heavy lined coat. The uniform added 25 pounds to his weight of 225 pounds.

The builder showed the inspector around the building. As part of his inspection, the inspector climbed on the window bars to evaluate this means of egress from the apartments on the upper floors in the event of a fire or emergency. When he reached the third floor, the painted window guards gave way, causing the inspector to fall and seriously hurt himself. Examination revealed that underneath the new paint, the window guards was badly corroded and could not support the inspector in his uniform. The inspector sued the builder for his injuries.

In a majority jurisdiction, who should prevail?

A. The inspector, because the builder received an economic benefit from his inspection of the building.

B. The inspector, because the builder had a duty to inspect the window bars.

C. The builder, because he did not intentionally conceal the defective bars with paint.

D. The builder, because he was not aware of the dangerous condition of the window bars.

A

D. The builder, because he was not aware of the dangerous condition of the window bars.

Discussion of correct answer: This is the best answer, because it correctly states the builder’s required standard of care toward the inspector. Although the builder invited the inspector onto the premises for the builder’s business purposes, courts traditionally classify the inspector as a licensee because he is a fireman. (This also applies to the police officer.) The builder’s duty to a licensee is to fix or warn of known dangers. He is not required to make his land safe for licensees and is not required to inspect to discover dangers, even if a reasonable inspection would have revealed them. If the builder was aware that the window bars would be dangerous to someone the inspector’s size, its condition would be a known defect about which the builder must issue a warning. Nothing in the facts shows that the developer knew of the unsafe condition.

How well did you know this?
1
Not at all
2
3
4
5
Perfectly
17
Q

An only daughter whose father died after a long illness arranged with a mortuary to bury him next to his wife’s grave. The daughter selected the most ornate casket available because she wanted it to remain closed during the wake, and she could not bring herself to view the body before the funeral. At the cemetery, however, she decided to view the body just before it was buried. She was horrified to discover that the body in the casket was dressed in a clown costume and a bright orange wig. In fact, it was not her father but a popular circus entertainer who had died the same day as her father and had requested to be buried in his costume. Although the mortuary was able to retrieve her father’s body and bury it, the daughter was greatly distressed by the episode and suffered nightmares as a result. However, she did not seek medical or psychiatric care because of it. The mortuary apologized for its error in switching the bodies, but insisted that the daughter pay all of the agreed-to charges for the funeral.

If the daughter brings action against the mortuary to recover for her emotional distress, can she recover damages?

A. No, because the daughter did not have to obtain medical or psychiatric care.

B. No, because the daughter suffered no physical injury.

C. Yes, because of the known sensitivity of people concerning the death of a family member.

D. Yes, because the mortuary is requiring the daughter to pay the bill for the funeral expenses.

A

C. Yes, because of the known sensitivity of people concerning the death of a family member.

The daughter can recover damages for her emotional distress, even though she suffered no physical injury and did not require medical care, because of the known sensitivity of people concerning the death of a family member. In the usual case, the duty to avoid negligent infliction of emotional distress is breached when defendant creates a foreseeable risk of physical injury to plaintiff, typically by causing a threat of physical impact that leads to emotional distress, and plaintiff can recover for physical injury caused solely by the distress. In special situations, however, courts have permitted plaintiff to recover in the absence of physical symptoms where defendant’s negligence creates a great likelihood of severe emotional distress. One of these situations is the mishandling of a relative’s corpse, because it is certainly foreseeable that a person will suffer severe emotional distress if the corpse of a family member is negligently mishandled. In this case, the mortuary was negligent in putting the wrong body in the casket, creating a foreseeable risk of severe emotional distress to the daughter under the circumstances. Despite the fact that she suffered no physical injury, she can recover damages from the mortuary.

How well did you know this?
1
Not at all
2
3
4
5
Perfectly
18
Q

A doorman negligently locked a door that an office worker was intending to use to exit an office building, so the worker was forced to use a different exit. As she stepped onto the sidewalk outside the building, a car careened out of control on the street and jumped the curb. The car struck and injured the worker and then drove off. The driver was not found.

The worker brought suit against the doorman, seeking damages for her injuries. At trial, the parties stipulated that the doorman was negligent in locking the door and that the worker suffered injuries when she was struck by the car. The worker also established that if she had exited from the door she was intending to, she would not have been struck by the car. At the end of the worker’s case, the doorman moved for a directed verdict in his favor.

How should the judge rule?

A. Grant the motion, because the driver of the car was the actual cause of the worker’s injuries.

B. Grant the motion, because the car was an unforeseeable intervening force.

C. Deny the motion, because the jury could find that but for the doorman’s negligence, the worker would not have been injured.

D. Deny the motion, because the jury could find that the doorman’s negligence was a foreseeable concurring cause of the worker’s injury.

A

B. Grant the motion, because the car was an unforeseeable intervening force.

The court should grant the motion because the evidence establishes that the car was a superseding force that cut off the doorman’s liability for his negligence under proximate cause principles. The general rule of proximate cause is that the defendant is liable for all harmful results that are the normal incidents of, and within the increased risk caused by, his acts. An indirect cause case is one where the facts indicate that a force came into motion after the time of defendant’s negligent act and combined with the negligent act to cause injury to the plaintiff. Whether an intervening force will cut off the defendant’s liability for the plaintiff’s injury and be deemed superseding is determined by foreseeability. Here, nothing in the facts suggests that a car jumping the curb was a foreseeable consequence of the doorman’s negligently locking the door. Hence, the judge should grant the motion because the worker has failed to establish the proximate cause element of his prima facie case.

How well did you know this?
1
Not at all
2
3
4
5
Perfectly
19
Q

A man on his bike was run off the road when the driver of a car, who was not paying attention to the road, drifted into the bicycle lane. The man ran into the guardrail, pinning his leg against his bike, but did
not fall. After the incident, the man chose not to go to a doctor, as he was a little shaken and sore, but otherwise did not appear injured. The next week, as the man was delivering a package as part of his job, his legs suddenly gave way, causing him to tumble down several flights of stairs. He was then rushed to the hospital, where he was treated for two broken legs and a shattered hip.

If the man now sues the driver of the car, which of the following statements is the most accurate?

(A) The driver is responsible for all injuries sustained by the man, because the driver must take the victim as he finds him.

(C) The driver is not responsible for all injuries sustained by the man, because the man failed to go to the doctor, who may have warned him to stay off of his feet.

(D) The driver is not responsible for all injuries sustained by the man, under the doctrine of avoidable consequences.

(B) The driver is responsible for all injuries sustained by the man, because the man’s conduct after the accident was reasonable.

A

(B) The driver is responsible for all injuries sustained by the man, because the man’s conduct after the accident was reasonable.

How well did you know this?
1
Not at all
2
3
4
5
Perfectly
20
Q

Two law students ranked high in their class were competing for one opening at a prestigious law firm. During the interview with the hiring partner, one student was asked what he thought of the other’s work as an editor of the law review. The student responded that there was a rumor around the school that the editor got outside help on her law review comment. Based in large part on his statement, that student was chosen over the law review editor, who later accepted a less lucrative position with another firm.

If the law review editor brings a slander action against the other student and establishes the above facts, will she prevail?

A. Yes, because the student’s statement to the hiring partner was defamatory.

B. Yes, because the law review editor suffered special damages.

C. No, because the hiring partner asked the student for his opinion.

D. No, because the law review editor did not establish that the student made the statement with at least negligence.

A

A. Yes, because the student’s statement to the hiring partner was defamatory.

The student’s statement constitutes slander per se and therefore the student will be liable. To establish a prima facie case for defamation, the following elements must be proved:

(i) defamatory language on the part of the defendant;
(ii) the defamatory language must be “of or concerning” the plaintiff (i.e., it must identify the plaintiff to a reasonable reader, listener, or viewer);
(iii) publication of the defamatory language by the defendant to a third person; and
(iv) damages to the reputation of the plaintiff.

Here, the student’s suggestion that the law review editor received outside help on an article she authored impeaches her integrity and legal skills. The defamatory language directly related to the editor. The publication requirement is satisfied because the student made the statement to the hiring partner. To recover damages for slander, special damages must be pleaded and proved unless the spoken defamation falls within one of four categories, characterized as slander per se. Hence, a defamatory statement adversely reflecting on the plaintiff’s abilities in his business, trade, or profession is actionable without pleading or proof of special damages. The student’s statement adversely reflected on the law review editor’s honesty and capability in her profession, and as such is slander per se.

How well did you know this?
1
Not at all
2
3
4
5
Perfectly
21
Q

While driving to the grocery store, a 45-year-old man was hit from behind by a car that was being negligently driven by a woman. At the hospital, the doctor indicated that the accident itself should only have caused minor injuries. However, the doctor determined that due to a childhood condition, the man’s bones did not fully form properly, and as a result, the injuries he suffered were more severe. In fact, it was determined that the man will have to be placed into traction for one year, and that he would never walk again.

To what damages will the man be entitled?

A. He will be entitled to just those damages that can be attributed to the car accident.

B. He will be entitled to all damages he sustained as a result of the accident, because a tortfeasor is liable for the full consequences of the injury she causes.

C. He will be entitled to damages that a normal person would have sustained in a similar accident.

D. He will be entitled to damages for the traction, but not the paralysis, because of the doctrine of avoidable consequences.

A

B. He will be entitled to all damages he sustained as a result of the accident, because a tortfeasor is liable for the full consequences of the injury she causes.

How well did you know this?
1
Not at all
2
3
4
5
Perfectly
22
Q

A ballplayer became ill soon after consuming sunflower seeds marketed by a farm products company. The package of seeds was inspected and foreign matter was discovered on the seeds.

If the ballplayer brings an action against the farm products company on the basis of strict tort liability, which of the following would be most helpful for the company to avoid liability?

A. The foreign matter on the seeds was a rare mold that could not be detected by tests commonly used for establishing that sunflower seeds are safe for human consumption.

B. The seeds were sold in their natural state, and had not been manufactured or processed by the farm products company in any way.

C. In answer to an interrogatory, the ballplayer has acknowledged that he has no evidence that his illness was caused by the foreign matter on the seeds.

D. Although marketed under the label of the farm products company, the seeds had been collected and packaged for distribution by another company and any foreign matter on the seeds was the other company’s fault.

A

C. In answer to an interrogatory, the ballplayer has acknowledged that he has no evidence that his illness was caused by the foreign matter on the seeds.

How well did you know this?
1
Not at all
2
3
4
5
Perfectly
23
Q

A patient needing ankle surgery signed standard consent forms and liability waivers covering the surgeon scheduled to perform the surgery. Two hours before the operation was scheduled to be performed, one of the surgeon’s patients was brought into the emergency room with numerous orthopedic injuries that required immediate attention. The surgeon requested the head of orthopedic surgery, who was the leading authority on ankle surgery, to perform the ankle surgery for him so he could go to the emergency room. By the time the head surgeon arrived in the operating room, the patient was already sedated. He performed the operation with his usual skill and the operation was a complete success.

If the patient sues the head surgeon for battery, will she likely prevail?

A. Yes, but she may be entitled only to nominal damages.

B. Yes, because the head surgeon is vicariously liable for the original surgeon’s obtaining a replacement without the patient’s consent.

C. No, because the head surgeon performed the operation competently and the patient suffered no harm.

D. No, because a reasonable person similarly situated would have consented to the operation.

A

A. Yes, but she may be entitled only to nominal damages.

The patient can establish a prima facie case for battery. The fact that the operation was a success and that she may not be able to prove actual damages will not bar her recovery.

The prima facie case for battery requires: (i) an act by defendant that brings about a harmful or offensive contact to plaintiff; (ii) intent on the part of defendant to do the act; and (iii) causation.

Here, the head surgeon’s performing the operation on the patient’s ankle would be harmful or offensive contact because the patient had selected another surgeon to perform the operation and did not consent to the head surgeon’s participating in any way. Even if evidence of her distress is not adequate to prove actual damages, she will still be entitled to a judgment in her favor and nominal damages, because damages is not an element of the prima facie case for battery.

How well did you know this?
1
Not at all
2
3
4
5
Perfectly
24
Q
A
How well did you know this?
1
Not at all
2
3
4
5
Perfectly
25
Q

A woman was shopping at a department store. She got on the store’s escalator on the third floor and was riding it to the fourth floor. The escalator came to a sudden stop, causing the woman to fall backwards. At the time the escalator suddenly stopped, the woman was not holding onto the handrail. She suffered several broken bones and injuries to her back. Apart from the sudden stop, nothing indicated that something was wrong with the escalator. A regular maintenance examination was performed on the escalator six days before the incident. After the incident, a maintenance specialist examined the escalator and found it to be in proper working order. The woman brought a negligence action for damages against the department store. At trial, the woman requested a jury instruction based on the doctrine of res ipsa loquitur. The trial judge gave the requested instruction, and the jury returned a verdict in favor of the woman. The department store has filed an appeal claiming the trial court erred by giving an instruction on res ipsa loquitur.

Will the store likely prevail on appeal?

A. Yes, because res ipsa loquitur would not be available because the woman was contributorily negligent.

B. Yes, because the woman will not be able to demonstrate that the sudden stopping of an escalator is an event which would not ordinarily occur in the absence of negligence.

C. No, because res ipsa loquitur is always available when there is unexplained negligence.

D. No, because the woman will be able to demonstrate that it is more likely than not that the store’s negligence caused the escalator to suddenly stop.

A

B. Yes, because the woman will not be able to demonstrate that the sudden stopping of an escalator is an event which would not ordinarily occur in the absence of negligence.

Discussion of correct answer: The doctrine of res ipsa loquitur is available to help a plaintiff establish the defendant breached a duty of care when the circumstances make it impossible for a plaintiff to determine exactly what the defendant did or failed to do that gave rise to the injuries. When the doctrine applies, a jury is permitted to infer negligence. For the doctrine to apply, a plaintiff must establish three things. First, the event that caused the injury would not occur in the absence of negligence. Second, it is more likely than not that the defendant’s negligence caused the event. Third, the plaintiff is not responsible for the event that caused the accident. While the woman will be able to show she was not responsible for the escalator suddenly stopping, she will not be able to prove either the first or second elements. The sudden stop of an escalator is unlikely to always be the result of negligence. Further, the facts demonstrate that the defendant was not negligent, having done an inspection six days before the accident and following the accident it was determined the escalator was in proper working order.

Think Like a Lawyer

Res ipsa loquitur is not applied often, because it requires a very special set of facts.

Step by Step Walkthrough

Step 1: A plaintiff must establish that three requisites are present in order to invoke the doctrine of res ipsa loquitur: (1) the event that caused the plaintiff’s injury was one which would not ordinarily occur in the absence of negligence; (2) it is more likely than not that it was the defendant’s negligence that was responsible for the injury-causing event; and (3) the plaintiff was not responsible for the event that caused injury.

Step 2: In this fact pattern, the woman cannot show that her injury would not ordinarily occur in the absence of negligence. It may have simply been an accident, considering all the inspections that were done. In addition, it is thus not more likely than not that the store’s negligence was responsible. Consequently, on appeal, the appellate court will likely find that the jury instruction on res ipsa loquitur should not have been given, and the jury award must be overturned.

Step 3: Select the answer stating that the store will win, because the woman will not be able to demonstrate that the sudden stopping of an escalator is an event which would not ordinarily occur in the absence of negligence.

Step 4: Ignore the answer choice regarding contributory negligence, which would not prevent a jury instruction on res ipsa loquitur.

Step 5: Ignore the answer choice stating that res ipsa loquitur is always available when there is unexplained negligence, because that is not the law.

Step 6: Finally, ignore the answer choice stating that the woman will be able to demonstrate that it is more likely than not that the store’s negligence caused the escalator to suddenly stop. This is so because the store has demonstrated that it met its duty of care.

How well did you know this?
1
Not at all
2
3
4
5
Perfectly
26
Q

Without doing a background check, an employer hires a delivery man to make deliveries in local residential neighborhoods. A background check of the delivery man would have revealed that he had a history of causing motor vehicle accidents by driving while under the influence of drugs and alcohol.

If the delivery man causes an accident while making a delivery because he is under the influence of drugs or alcohol, can the injured party bring a cause of action based on negligent hiring against the employer?

A. Yes, because the cause of action will be based on the doctrine of respondeat superior.

B. Yes, because the employer was negligent in failing to conduct a background check on the delivery man before hiring him.

C. No, because the employer did not have a duty to investigate the delivery driver’s background before hiring him.

D. No, because the doctrine of respondeat superior does not apply to intentional torts committed by the employee.

A

B. Yes, because the employer was negligent in failing to conduct a background check on the delivery man before hiring him.

Discussion of correct answer: This question requires the reader to distinguish between the doctrine of respondeat superior and negligence in hiring.

The defendant-employer has a duty to a plaintiff to exercise reasonable care in hiring employees, such that a defendant may be liable to a plaintiff if an employee subsequently injures the plaintiff.

This is negligence in hiring.

On the other hand, the vicarious liability of an employer for his employee’s torts — respondeat superior – applies where the negligent conduct is that of the employee and liability is attributed by law to the employer.

In negligent hiring, the employer is liable for his own negligence in hiring the employee, not vicariously liable for the wrongful conduct of the employee. Here, the question specifically asks about negligence in hiring. Therefore, the reader should focus on the employer’s failure to conduct a background check, which would have revealed the delivery man’s history of causing motor vehicle accidents while under the influence of drugs and alcohol.

How well did you know this?
1
Not at all
2
3
4
5
Perfectly
27
Q

A woman went shopping for a birthday present for her husband at the local mall. The mall has a lighted parking area but there is no security staff that patrols the parking lot. The mall is located in an area with a high crime rate. As the woman was getting into her car, a criminal grabbed her purse and placed a gun to her head. The criminal threatened to shoot her if she did not give him her shopping bags. She resisted and he hit her with the gun before fleeing with her purse and the presents she bought for her husband.

In a negligence suit for damages against the mall, will the woman likely prevail?

A. Yes, because it was foreseeable that in a high crime area criminals could pose a danger to customers going to their vehicles in the parking lot.

B. Yes, because the mall would be strictly liable because of its knowledge of crime in the area.

C. No, because the negligence of the mall was not the proximate cause of the woman’s injuries.

D. No, because the mall satisfied its duty to exercise reasonable care for the safety of invitees by having a lighted parking lot.

A

A. Yes, because it was foreseeable that in a high crime area criminals could pose a danger to customers going to their vehicles in the parking lot.

Discussion of correct answer: Generally, the criminal or tortious acts of a third party constitute a superseding intervening cause that breaks the chain of causation. However, when the facts and circumstances demonstrate that the criminal act is foreseeable, the chain of causation will not be broken. The facts indicate that the mall is located in a high crime area and the mall does not use security guards. Therefore, these facts and circumstances indicate that a criminal act is foreseeable.

How well did you know this?
1
Not at all
2
3
4
5
Perfectly
28
Q

A retired policeman was hired as a night watchman by a manufacturer of large construction machinery. During the watchman’s orientation, the manager of the factory emphasized that in the event of any problem, the watchman was to keep himself out of danger and call the police immediately. The night watchman also was warned never to go outside the fence while on duty. One night, a drug addict, whom the watchman recognized from his days on the police force, was walking past the yard. In a rare moment of lucidity, the addict also recognized the former police officer. The addict jeered, “Hey! You a rent-a-cop now?” The watchman ignored the addict for a while, but the addict continued to rant loudly and derisively about the watchman’s new career. After several minutes of this, the watchman flew into a rage. He went outside the gate and chased the addict down. The watchman punched the addict several times, giving him a concussion. The addict subsequently filed a lawsuit against the watchman and the manufacturer.

Which of the following is the strongest argument in the manufacturer’s defense?

A The watchman’s act was outside the scope of his employment.
B The watchman was defending the manufacturer’s property.
C The watchman was acting in self-defense.
D The watchman was contributorily negligent.

A

A The watchman’s act was outside the scope of his employment.

How well did you know this?
1
Not at all
2
3
4
5
Perfectly
29
Q

While walking down an escalator in a department store, a man was hit from behind by an employee who was taking some boxes to the trash compactor. The man fell and suffered injuries as a result. While in the hospital, the doctor indicated that the accident itself should only have caused minor injuries. The doctor then determined the man’s bones had never healed properly after falling out of a tree as a child and breaking his pelvis and both of his legs. As a result, the injuries the man now suffered were more severe. At trial, the jury made the following findings: (1) the man was 10% at fault; (2) the employee was gainfully employed by the
department store at the time of the injury; and (3) the man’s childhood injuries were self-inflicted.

To what damages will the man be entitled?

(A) He will be entitled to only those damages that can be attributed to the employee’s accident, minus his own percentage of negligence.

(B) He will be entitled to all damages sustained as a result of the accident minus his own percentage of negligence, because the department store is liable for the full consequences of the injuries sustained by the man.

(C) He will be entitled to damages that a normal person would have sustained in a similar accident.

(D) He will recover nothing, because his childhood injuries were self-inflicted.

A

(B) He will be entitled to all damages sustained as a result of the accident minus his own percentage of negligence because the department store is liable for the full consequences of the injuries sustained by the man.

How well did you know this?
1
Not at all
2
3
4
5
Perfectly
30
Q

A traffic accident occurred at a road intersection. A motorcycle, a car, and a truck were involved. The motorcyclist
was injured and brought suit against the driver of the car and the driver of the truck. The jury returned a verdict finding that the motorcyclist’s injuries were caused by negligence on the part of all three of the parties.
The jury assigned 55% of the fault for the motorcyclist’s injuries to the motorcyclist, 25% to the driver of the car, and 20% to the driver of the truck. The jury found that the number of the motorcyclist’s injuries was $100,000. The motorcyclist enforced the judgment against the driver of the car and collects $45,000 from him. The driver of the car then brought an action against the driver of the truck for contribution.

What should the driver of the car recover?

(C) $20,000.
(B) $27,000.
(A) Nothing, because he was more at fault than the driver of the truck was.
(D) $15,000.

A

(C) $20,000.

How well did you know this?
1
Not at all
2
3
4
5
Perfectly
31
Q

A husband and a wife were always misplacing their keys. One day, the husband hung a key rack at eye level just inside the front door of their home. The husband and the wife agreed that each night, when they returned home from work, they would place their keys on the key rack so that each knew at all times where their keys were located.

One Sunday afternoon, the husband and the wife decided to walk to the local deli for lunch. Because their 14-year-old daughter was going to be home, they decided they did not need to take their keys, and so they left them hanging on the key rack. Shortly after the husband and the wife left, the daughter decided that she wanted to attempt to drive the family car around the block before her parents returned. Even though she knew she was forbidden from doing so, she removed the keys to the family car off of the key rack, started the family car, and attempted to drive it around the block. However, due to the fact that she was an inexperienced driver, she quickly lost control of the car and crashed into a parked car owned by a neighbor. The car suffered substantial damage.

If the neighbor sues the husband and the wife, what will be the most likely result?

A. The neighbor will win, because the husband and the wife are liable for the negligence of their daughter.

B. The neighbor will win, because a child operating a motor vehicle is an unusually dangerous activity.

C. The husband and the wife will win, because a parent is not liable for the tortious acts of a child.

D. The husband and the wife will win, unless they did not act reasonably in permitting their daughter access to the car keys.

A

D. The husband and the wife will win, unless they did not act reasonably in permitting their daughter access to the car keys.

Think Like a Lawyer

Parents are not liable for the negligent acts of their children.

Step by Step Walkthrough

Step 1: While a child may be negligent if he does not act as the reasonable child of the same age, education, intelligence, and experience would have done, his negligence is not attributable to his parents. Parents are not vicariously liable for negligence committed by their child. However, parents can be liable for their own negligence if they fail to act as reasonable prudent parents under the circumstances.

Step 2: Here, the 14-year-old is not a defendant. The parents are defendants, and they will only be liable if they breached a duty of reasonable care in regard to the car keys.

Step 3: Select the answer stating that, if the neighbor sues the husband and the wife, the husband and the wife will win, unless they did not act reasonably in permitting their daughter access to the car keys.

Step 4: Discard the answer choice saying that the neighbor will win, because the husband and the wife are liable for the negligence of their daughter. This is not a correct statement of the law. There is no vicarious liability on the part of the parents for the negligent acts of their children.

Step 5: Discard the answer choice saying that the neighbor will win, because a child operating a motor vehicle is an unusually dangerous activity. Driving a car is not ultrahazardous, and strict liability does not apply. Instead, the 14-year-old would be held to the standard of an adult driving the car, if she were a defendant. The parents, since strict liability is not involved, have no liability for their child’s negligent act.

Step 6: Finally, discard the answer choice saying that the husband and the wife will win, because a parent is not liable for the tortious acts of a child. This is true, but incomplete, because there is one theory left in which the parents might be liable, and that is for their own negligence.

How well did you know this?
1
Not at all
2
3
4
5
Perfectly
32
Q

Driving home from work one evening, a man suddenly had engine trouble. Fearing that he would permanently damage his engine, the man immediately pulled to the side of the road and parked. The man failed to notice he had parked five feet from a fire hydrant. The man got out of his car to call for help. Shortly thereafter, a car negligently crashed into the man’s parked car. A local city ordinance made it illegal to park within 10 feet of a fire hydrant. The man sued the other driver for the damage to his car.

What result is most likely?

A Judgment for the other driver, because the man’s violation of the city ordinance constituted comparative negligence per se. B Judgment for the other driver, because the man’s parking his car next to the fire hydrant was the cause in fact of the accident. C Judgment for the man, because the city ordinance was designed to provide the fire department access to the fire hydrant. D Judgment for the man, because the other driver had the last clear chance to avoid the accident.

A

C. Judgment for the man, because the city ordinance was designed to provide the fire department access to the fire hydrant.

How well did you know this?
1
Not at all
2
3
4
5
Perfectly
33
Q

A man bought a brand new microware oven from a department store. This new microware was advertised as being state of the art and could cook meat quickly and without a lot of hassle. The man invited some of his friends over to try it out. The man put some food in the microwave. As the man and his friends talked to each other, they smelled smoke. The man opened up the new microware and was severely injured as it was on fire. It turned out that the new microware had a manufacturing defect and one of the faulty wires caused it to catch on fire. If the microware had been manufactured correctly, the fire would not have happened. The man was very angry and wanted to bring a strict liability claim to recover money for his injuries.

Who can the man bring the claim against?

A. The department store.

B. The manufacturer of the product.

C. Both the department store and the manufacturer of the product.

D. Neither the department store nor the manufacturer of the product.

A

C. Both the department store and the manufacturer of the product.

How well did you know this?
1
Not at all
2
3
4
5
Perfectly
34
Q

A father and his son went camping in a very rural area. They camped at the bottom of a steep hill covered with large trees. During the first night of camping, a large tree limb fell and seriously injured the father. The father and son were about five miles from a public highway. The son decided to walk to the highway to get medical help. At the top of the hill, the son came upon two experienced campers. The campers and the son decided he would continue walking to the highway to get help, and the two campers would stay with his father at the bottom of the hill until help arrived. After finding the father, the campers decided to move the father to the top of the hill and wait for help to arrive. As they were moving the father up the hillside, they lost their footing and dropped the father. The father rolled about 50 feet back down the hill. Eventually, help arrived and the father was taken to a hospital. The treating physician determined that moving the father and dropping him worsened his internal bleeding and caused more severe injuries.

In a negligence suit against the campers, will the father prevail?

A. Yes, because the rescuers were subject to a heightened standard of care.

B. Yes, because the rescuers did not act reasonably.

C. No, because the rescuers acted reasonably.

D. No, because the rescuers will be immune from liability under a Good Samaritan statute.

A

B. Yes, because the rescuers did not act reasonably.

Discussion of correct answer: While a person generally has no duty to take affirmative steps to render assistance absent a special relationship, in a majority of jurisdictions once action is taken the rescuer is held to a standard of reasonable care. Thus, a rescuer must act with due care in carrying out a rescue. When a special relationship exists, such as parent-child, employer-employee, or jailer-prisoner, the defendant has an affirmative duty to act to benefit the plaintiff. Here, there is no special relationship. However, the facts show that the two campers did not act reasonably when they decided to move the injured father up the hill, placing him at even greater risk of injury. A reasonable rescuer would not have attempted to move an injured person under those conditions.

How well did you know this?
1
Not at all
2
3
4
5
Perfectly
35
Q

A manufacturer of component parts supplied pedals to a large bicycle producer. One of the pedals ended up being defective, but was still placed on a bicycle, because the producer was not aware that the pedal was defective. The bicycle was later sold to a father, who bought it for his son. Two weeks after the purchase of the bicycle, the defective pedal broke when the son was riding his bicycle at a local park. As a result, the boy crashed into an elderly man, who suffered serious injuries. The elderly man filed a strict product liability action against the bicycle producer.

Will the elderly man prevail in his lawsuit?

A. Yes, because the bicycle producer is in the chain of distribution.

B. Yes, because of the consumer expectation test standard.

C. No, because the defect is the fault of the manufacturer of component parts.

D. No, because the elderly man was not using the defective product.

A

A. Yes, because the bicycle producer is in the chain of distribution.

How well did you know this?
1
Not at all
2
3
4
5
Perfectly
36
Q

A young man had decided to move from his house into an apartment he just bought in the city. The apartment was much smaller than the house, but it was in a very nice building. The young man decided to post an advertisement on-line in order to sell one of the couches that he would not be taking to the new apartment. The couch was very large and had very high legs. The young man knew that one of the legs of the couch was broken and was only taped to the couch, but the young man did not mention that in the advertisement. At some point a woman contacted the young man and bought the couch for $500. The woman then set the couch in her house. A week later, the woman’s elderly grandmother sat on the couch, but the broken leg fell off, and the couch collapsed with the grandmother. As a result, the grandmother suffered a hip injury. The grandmother filed a strict products liability suit against the young man for her injury.

Which of the following is correct?

A. The grandmother prevails, because the young man did not warn the woman about the broken leg.

B. The grandmother prevails, because the couch had a broken leg.

C. The young man prevails, because the grandmother was not the purchaser of the couch.

D. The young man prevails, because he was a one-time seller.

A

D. The young man prevails, because he was a one-time seller.

How well did you know this?
1
Not at all
2
3
4
5
Perfectly
37
Q

A local hair salon was busy and filled with both customers and employees. At this salon, customers had to walk down an aisle to get their hair washed before returning to the stylists’ chairs. A stylist placed a bottle of conditioner on the floor next to her chair because her work counter was full. The stylist accidentally knocked the bottle over, but did not notice what she had done. Moments later, a customer walking down the aisle slipped on the liquid that had spilled onto the floor. A sign at the front of the store warned customers to watch their step. The customer has filed a suit for damages against the store.

Will the store likely prevail?

A. Yes, because the store discharged its duty by warning shoppers to watch their step.

B. Yes, because the store discharged its duty to maintain the premises in a reasonably safe condition.

C. No, because the store’s duty was not discharged by the giving of a warning and it failed to maintain the premises in a reasonably safe condition.

D. No, because the customer was at fault for not seeing the danger.

A

C. No, because the store’s duty was not discharged by the giving of a warning and it failed to maintain the premises in a reasonably safe condition.

Discussion of correct answer:The standard of care that applies to a landowner or occupier depends on the status of the injured party and the type of activity that caused the injury. Under these facts, the customer would be a business invitee because she entered the premises at the express or implied invitation of the landowner. She entered for a specific purpose relating to the store’s activities on the land. A landowner must exercise reasonable care to prevent injuries to an invitee from activities conducted on the land. The landowner also has a duty to exercise reasonable care in discovering dangerous artificial conditions that an invitee would not be aware of and to warn them of the condition or to make it safe. Merely providing the customer a warning would not satisfy the store’s duty to prevent injuries or its duty to discover dangerous conditions

How well did you know this?
1
Not at all
2
3
4
5
Perfectly
38
Q

An experienced mechanic was hired by a company to replace a defective ignition switch on a motorized three-wheel forklift. The forklift had been purchased by the company two years ago from a distributor of heavy-duty factory equipment. The ignition switch that the mechanic was working on was located near the two rear wheels. After removing the old defective switch, the mechanic then attempted to insert the new ignition switch, which he had purchased from a parts distributor. When the new ignition switch popped out of the socket, the mechanic tried to reinsert it. As he did so, the forklift engine suddenly started and dragged the mechanic underneath the rear axle, injuring him. The new ignition switch that the mechanic was installing had been specifically designed to fit the model he was fixing.

If the mechanic brings a claim against the manufacturer alleging that the new ignition switch was defective and unreasonably dangerous, will he recover?

A. Yes, if such an accident was foreseeable and the manufacturer failed to warn purchasers of the danger.

B. Yes, if the manufacturer advertised the ignition switch as “a safe and effective starter switch” for such forklift models.

C. No, if a reasonably prudent mechanic would have stopped trying to reinsert the ignition switch after it had popped out of the socket.

D. No, if the mechanic failed to properly inspect the ignition switch before attempting to insert it.

A

A. Yes, if such an accident was foreseeable and the manufacturer failed to warn purchasers of the danger.

How well did you know this?
1
Not at all
2
3
4
5
Perfectly
39
Q

A pedestrian, who was walking along Chestnut Street at 10:20 p.m. on the night of December 3, urgently needed to find a restroom. Just ahead, the pedestrian noticed a private dinner club. As the pedestrian
approached the club, he noticed a sign over the front door that read: “ADMITTANCE TO MEMBERS ONLY.” Although the pedestrian was not a member of the exclusive club, he entered the dimly lit club, found a door marked “Gentlemen,” and entered the restroom.

Which of the following would best describe the pedestrian’s legal status when he was in the restroom?

(C) Licensee.
(B) Guest.
(D) Invitee.
(A) Trespasser.

A

(A) Trespasser.

How well did you know this?
1
Not at all
2
3
4
5
Perfectly
40
Q

A statute in State X provides that:

“It shall be unlawful for any person to sell, or permit to be sold, intoxicating liquors to one who visibly is under the influence of alcohol. Whosoever shall sell intoxicating liquors in contravention of the foregoing provision shall be guilty of a misdemeanor punishable by imprisonment for not more than 60 days or a fine of not more than $5,000 or both.”

The owner of a local bar and grill brought the statute to the attention of his bartenders. Late one night, a patron entered the bar and ordered a gin martini. Noticing that the patron was very drunk, the bartender said, “Hey buddy, I don’t think you can handle another drink.” The patron responded, “C’mon, just one more for the road.” The bartender acquiesced and mixed a martini from a bottle of gin, which he then served to the patron.

The bartender was unaware that a disgruntled employee of the gin distiller had spiked the bottle with strychnine, a highly poisonous substance. After drinking the martini, the patron left the bar and began to drive home. Moments later, due to the poison he became violently ill and went into convulsions. As a consequence, the patron lost control of his car, which struck and seriously injured a pedestrian. The patron, who subsequently recovered, was prosecuted under a state statute making it a misdemeanor to drive while under the influence of alcohol.

If the pedestrian asserts a claim against the bartender, will the bartender likely be held liable?

A. No, because the pedestrian was not a foreseeable plaintiff.

B. No, because the bartender’s act was not the legal cause of the accident.

C. Yes, because he was negligent per se in serving the patron the martini.

D. Yes, because but for the fact that he served the patron the martini, the pedestrian would not have been injured.

A

A. No, because the pedestrian was not a foreseeable plaintiff.

Think Like a Lawyer

Causation requires both actual cause and proximate cause.

Step by Step Walkthrough

Step 1: Negligence requires: (1) duty; (2) breach of that duty; (3) causation, both actual and proximate; and (4) damages. Actual cause means the cause-in-fact of the injury. Legal cause, also known as proximate cause, means whether the defendant owed a duty of reasonable care to the plaintiff as one who could foreseeably be injured.

Step 2: The violation of a safety statute, or other criminal statute, can provide the first two elements, leaving only causation and damages to be proved. This is known as negligence per se. The violation of a safety statute can prove duty and breach if the harm suffered is the harm that the statute was meant to protect against, and the plaintiff is a member of the protected class.

Step 3: Here, the bartender acted negligently in serving alcohol to the patron that was already drunk. In addition, the bartender violated the dramshop statute that had been enacted. The harm that occurred, however, was not one that the statute addressed. Driving under the influence of strychnine was not a situation that the statute was designed to cover.

Step 4: Therefore, if the pedestrian asserts a claim against the bartender, the bartender will not be liable, because the bartender’s act was not the legal cause of the accident. Select this answer. The bartender did not foresee that a pedestrian could be injured by strychnine, because he didn’t know he was adding the strychnine to the drink. The two persons responsible for the accident were the patron and the saboteur of the gin.

Step 5: Ignore the three incorrect answer choices. The pedestrian is a proper, foreseeable plaintiff. There was no negligence per se. And, finally, it is true that, but for the fact that he served the patron the martini, the pedestrian would not have been injured. Thus, the serving was a cause-in-fact,but it was not the proximate cause of the accident. The bartender could not have assessed the foreseeability of a strychnine-related accident, because he had no idea strychnine was in the gin.

How well did you know this?
1
Not at all
2
3
4
5
Perfectly
41
Q

A local elementary school was located on the west side of a street. The school was totally obscured from the roadway by a tall hedge designed to shield out road noise and give the students a sense of rural serenity. An opening in the hedge led to a crosswalk which the elementary school students often used. In addition, there were no signs visible on the street indicating the existence of the elementary school. A tourist was proceeding north on this street at 25 mph, the normal speed limit within city limits, when an elementary school student emerged from the opening in the hedge and ran into the street. The tourist hit the student, critically injuring him. Although no sign was posted, another local ordinance set a 15-mph speed limit for streets surrounding any school. If the tourist had been going the 15-mph speed limit, he would have been able to stop in time to avoid hitting the student.

If the student’s mother asserts a claim against the tourist on the student’s behalf, will the student prevail?

A. No, because the hedges prevented a driver on the street from seeing the school.

B. No, unless the tourist should have been driving more slowly under the circumstances.

C. Yes, because the tourist was negligent per se.

D. Yes, because the tourist owed the student a duty of reasonable care.

A

A. No, because the hedges prevented a driver on the street from seeing the school.

Discussion of correct answer: The tourist violated a local ordinance requiring drivers to maintain a 15-mph speed limit on streets surrounding a school. The purpose of the statute is to protect school students, be they adult or otherwise, from serious injury from rapidly moving cars. However, violation of a statute only proves breach when it is an unexcused violation. Here, the tourist was not a local resident, the existence of the school was hidden by the hedge, and no sign existed informing the tourist that this was a school district. The tourist did not know that he faced a situation in which the statute would apply. A reasonable person in his situation would not have known to slow down to the 15-mph limit. Hence, the tourist’s noncompliance will be excused if he was driving as a reasonable person under the circumstances. Keep in mind that ignorance of the law is no defense to a crime, but here we are not dealing with criminal liability, but rather civil tort liability.

Think Like a Lawyer

In order for a person to comply with a safety statute regarding behavior around a school, there must be notice of some kind that a school exists, either by signage or by being able to actually see the school.

Step by Step Walkthrough

Step 1: Negligence requires: (1) duty; (2) breach of that duty; (3) causation, both actual and legal; and (4) damages. The violation of a safety statute, or other criminal statute, can provide the first two elements, leaving only causation and damages to be proved. This is known as negligence per se. The violation of a safety statute can prove duty and breach if the harm suffered is the harm that the statute was meant to protect against, and the plaintiff is a member of the protected class. However, the violation will be excused if compliance with the statute: (a) would have resulted in a greater harm; or (b) would have been impossible.

Step 2: Here, the tourist drove at 25 mph, which was within the duty of care for a driver within the city limits. Although there was a statute setting a lower speed limit around schools, there was no sign about the school. Thus, the tourist could not have known to drive more slowly.

Step 3: Select the answer stating that the student will not prevail, unless the tourist should have been driving more slowly under the circumstances. This would be establishing ordinary negligence.

Step 4: Ignore the answer choice saying that the student will not prevail, because the hedges prevented a driver on the street from seeing the school. Even if the hedges hid the school, the driver still might be liable on an ordinary negligence theory.

Step 5: Ignore the answer choice saying that the student will prevail, because the tourist was negligent per se. The tourist was not negligent per se. Although the statute was violated, and the harm that occurred was the harm that the statute was designed to protect against, there was no notice that a school existed. The tourist is not required to be psychic. In order for an ordinary person to comply with such a safety statute, there must be notice of some kind that a school exists, either by signage or by visual information.

Step 6: Finally, ignore the answer choice saying that the student will prevail, because the tourist owed the student a duty of reasonable care. It appears, under the facts as stated, that the tourist met the duty of reasonable care. Thus, this answer choice does not help the student.

How well did you know this?
1
Not at all
2
3
4
5
Perfectly
42
Q

A leading manufacturer of camping products marketed a thermal blanket that incorporated new heat-generating technology. On the blanket’s packaging was a label stating that it was recommended by a leading camping association. A camper purchased one of the blankets and took it with her on a fall camping trip. The first night that she used the blanket, it malfunctioned and caused her severe burns.

If the camper maintains an action against the camping association, on which theory is she most likely to recover, if at all?

A. Express warranty.

B. Implied warranty of merchantability.

C. Strict tort liability.

D. Negligence.

A

D. Negligence.

How well did you know this?
1
Not at all
2
3
4
5
Perfectly
43
Q

An absentee landlord owned a run-down apartment building. He made as few repairs as possible. One day, a man visited the landlord’s building to see whether he wanted to rent an apartment there. The landlord showed the man the top-floor apartment that was available. The landlord quickly steered the man into the apartment, hoping that the man wouldn’t notice that the front door was about to come loose and fall off. The man did not in fact notice the broken door. The man nevertheless decided to rent the apartment, and moved in that Sunday night. The next morning, the tenant in the apartment below the man’s heard a loud crash from upstairs. The tenant raced upstairs to find the man on the floor, unconscious. The broken door was lying on the floor near the man. The man sued the landlord for negligence.

If the man prevails, which of the following provides the best reason?

A The door represented a patent defect.
B The door represented a latent defect.
C The contract did not contain any language that stated the man was renting the property “as is.”
D The landlord breached the implied warranty of habitability.

A

B The door represented a latent defect.

A landlord has a duty to warn a tenant of latent defects or repair them. Generally, a landlord is under no duty to warn of or repair obvious conditions. However, a landlord is responsible for dangerous natural or artificial conditions of which:

(1) the tenant is unaware and the condition is not reasonably apparent; and
(2) the landlord is or should reasonably be aware. Here, the landlord was aware of the broken door, and did nothing to fix.

How well did you know this?
1
Not at all
2
3
4
5
Perfectly
44
Q

As the holiday season approached, a homeowner decided to hang lights around his home. He set up a ladder, climbed it, and began to hang the lights along the edge of his roof. As the homeowner reached the corner of his roof he attempted to stretch to attach the lights to the corner rather than reposition the ladder. In so doing, he fell about 12 feet off the ladder. The homeowner felt some pain in his lower back but did not believe he was seriously injured. Shortly thereafter, the homeowner decided he needed more lights to hang on his home. While driving to the store to purchase additional lights, the homeowner purchased a hamburger and began to eat it his car while driving when he was hit from the rear by another vehicle, that had negligently failed to stop. As a result of the collision the homeowner suffered paralysis. The homeowner brought suit against the other driver. At trial, medical experts testified that the homeowner would not have been paralyzed in the collision if he had not fallen off the ladder earlier that day and that the initial back injury could have been treated by any competent practitioner by immobilization in a brace if the homeowner had gone to the doctor immediately.

What is the driver’s best argument for a reduction in the damages owed to the homeowner?

A. Damages should be reduced, because of the doctrine of avoidable consequences.

B. Damages should be reduced, because the driver was only passively negligent.

C. Damages should be reduced, because the homeowner was comparatively negligent.

D. Damages should be reduced, because the driver was not the legal cause of the homeowner’s injury.

A

C. Damages should be reduced, because the homeowner was comparatively negligent.

Discussion of correct answer:This question requires that a distinction be drawn between the doctrines of avoidable consequences and comparative negligence. When a plaintiff is injured in an accident, a plaintiff has an obligation to mitigate his damages. This includes taking all reasonable steps to limit the adverse consequences of the accident. If the plaintiff does not act as a reasonable person in limiting his injuries, his recovery against the individual causing the incident will not include damages for those consequences that could have been avoided. In this case the homeowner did not fail to mitigate damages after the tort, so the concept of avoidable consequences does not apply. Instead, he contributed to his injury because he was contributorily negligent by eating a hamburger while driving, which would make him comparatively negligent.

How well did you know this?
1
Not at all
2
3
4
5
Perfectly
45
Q

Several cars of a freight train transporting nuclear waste derailed as the train neared a street crossing. One of the cars struck a motorist’s car as it was waiting at the crossing gate, seriously injuring the motorist. The area around the accident was immediately evacuated, but fortunately none of the freight cars ruptured in the derailment.

In an action alleging strict liability against the railway that operated the freight train, the motorist established the above facts and presented evidence of her injuries. The railway presented evidence that the derailment was caused by a hidden defect in the spikes that anchored the rails to the track. The spikes were manufactured by its regular supplier and had not previously caused any problems. The railroad also presented evidence that the local authorities were supposed to restrict access to roads crossing the freight line while that particular train was in transit, but they had failed to do so.

In this action, is the motorist likely to prevail?

A. Yes, because the spikes were in a defective condition that made them unreasonably dangerous.

B. Yes, because the railway was engaged in an abnormally dangerous activity.

C. No, because the injury did not arise from the dangerous propensity of the activity.

D. No, because the negligence of the local authorities in failing to restrict access to roads crossing the freight line was a superseding cause of the motorist’s injuries.

A

C, No, because the injury did not arise from the dangerous propensity of the activity.

The motorist is not likely to prevail in a strict liability action because her injury did not arise from the abnormally dangerous propensity of the railway’s activity. The railway’s transport of nuclear waste likely qualifies as an abnormally dangerous activity because: (i) it creates a foreseeable risk of serious harm even when reasonable care is exercised by all actors; and (ii) the activity is not a matter of common usage in the community. However, the scope of liability extends only to the dangers that would be anticipated from the activity involved; strict liability does not apply to harms that were not caused by the normally dangerous propensity of the activity. Here, the railway’s activity is subject to strict liability because of the danger of radioactivity inherent in nuclear waste, but not from a derailment by itself. Because the motorist’s injuries were not caused by the release of radioactivity, strict liability does not apply. The railway would be liable for the injuries from the derailment only if the motorist established negligence.

How well did you know this?
1
Not at all
2
3
4
5
Perfectly
46
Q

An office assistant leased a used car from a car dealer. While driving the car, which was a gas/electric hybrid, the office assistant lost control of the car in a driving rain storm and crashed head-on into a tree. An emergency crew was dispatched in response to a call from a witness who said that the driver of the car was injured, but the witness was unable to open the car’s door. The emergency crew arrived and used specialized hydraulic equipment to cut through the car’s door. Unfortunately, although it did not make a sound, the hybrid car was still running and the operator of the hydraulic equipment received a nasty electric shock that sent him to the hospital. It was later determined that the car model had a design defect.

Does the emergency worker have a product liability action against the car dealership that leased the car?

A. No, because a commercial lessor is not liable for the lease of defective used goods.

B. No, because the emergency worker was not the consumer of the product and is therefore not a proper defendant.

C. Yes, because the emergency worker is a reasonably foreseeable plaintiff.

D. Yes, because the car dealership was negligent in leasing a defective car.

A

C. Yes, because the emergency worker is a reasonably foreseeable plaintiff.

How well did you know this?
1
Not at all
2
3
4
5
Perfectly
47
Q

A thief made his living by stealing other people’s cattle and selling the meat to unscrupulous butchers. He drove his truck and trailer to a rancher’s ranch one evening because he had heard that the rancher was famous for his cattle. The thief set up his trailer and climbed into a field intending to steal a cow. Unknown to the thief, the field contained the rancher’s prize fighting bull, used in bullfighting. The bull charged the thief and knocked him to the ground, breaking his ankle in the process. A driver who happened to be driving by the rancher’s property saw the thief fall and, knowing of the rancher’s bull, realized that the thief was in extreme danger. The driver stopped his truck, got out and vaulted the fence into the field. Waving a red bandanna to distract the bull, the driver shouted to the thief to get out of the field while the driver would draw the bull off. The bull charged the driver, who spent an exciting few minutes dodging the enraged animal while the thief hobbled to the fence and climbed over. As the driver abandoned his bandanna and leaped over the fence, his foot caught on the boards and he fell out of the field, breaking his leg.

The driver sued the thief for his injuries. What result?

A. Judgment for the thief, because the driver knew that the bull was dangerous and voluntarily exposed himself to the danger.

B. Judgment for the thief, because the driver’s decision to intervene was an abnormal response to the situation.

C. Judgment for the driver, because he prevented the thief from suffering almost certain death or severe injury from the bull.

D. Judgment for the driver, because the thief negligently or intentionally placed himself in peril.

A

D. Judgment for the driver, because the thief negligently or intentionally placed himself in peril.

How well did you know this?
1
Not at all
2
3
4
5
Perfectly
48
Q

The plaintiff purchased a small, decorative glass table from her favorite home furniture and decoration store. The table is very small and the glass that makes up the surface of the table is very thin. The store displayed the table in its “decorations” section. When the plaintiff brought the table home, she placed it in her living room. A few days later, when the plaintiff came home from work, she lightly tossed her purse on the table. The table shattered, and the plaintiff was injured by shards of glass.

If the plaintiff brings an action against the store under a products liability theory, will she succeed?

A. Yes, because the defense of misuse is not available.

B. Yes, because it was foreseeable the plaintiff would use the table for this purpose.

C. No, because the manner in which the plaintiff used the table was neither intended nor foreseeable.

D. No, because the plaintiff misused the table.

A

B. Yes, because it was foreseeable the plaintiff would use the table for this purpose.

How well did you know this?
1
Not at all
2
3
4
5
Perfectly
49
Q

A man was driving home one evening when he looked away from the road to change the channel on the car radio. Because he looked away from the road he failed to notice that a pedestrian was lawfully crossing the street. When the man looked back and saw the pedestrian, he slammed on his brakes, but was unable to stop in time to avoid hitting the pedestrian. However, the collision was minor and the pedestrian, while knocked to the ground and momentarily dazed, suffered no permanent injuries.

Almost immediately, another car came driving down the road. The other driver negligently failed to see the man’s stopped car and plowed into the back of it. This pushed the man’s car over the pedestrian’s leg and he suffered a broken leg as a result. The pedestrian sues the man and the other driver for his injuries.

Will he prevail against the man?

A. Yes, because the man’s negligence made a significant contribution to the pedestrian’s injury.

B. Yes, because both the man and the other driver were negligent.

C. No, because the other driver’s subsequent impact would have caused the injury anyway.

D. No, because the other driver’s act was a superseding force.

A

A. Yes, because the man’s negligence made a significant contribution to the pedestrian’s injury.

Discussion of correct answer: A defendant’s conduct is the cause-in-fact of the plaintiff’s injury if that conduct was a substantial factor in causing the injury. Each driver made a significant contribution to the injury suffered by the pedestrian. The man’s significant contribution was that he knocked the pedestrian down on the ground. When there are multiple negligent parties, each of whom contributed to the plaintiff’s indivisible harm, they will be jointly and severally liable. Because the harm caused here is indivisible, both the man and the driver are jointly and severally liable for the pedestrian’s injuries.

How well did you know this?
1
Not at all
2
3
4
5
Perfectly
50
Q

An investor bought a run-down house in a developing neighborhood, which he hoped to fix up and “flip” for a considerable profit. The investor hired a painter to paint the outside of the house. The painter was a friend of the investor’s who was trying to start his own painting company. The painter agreed to paint the house at a discounted rate if the investor would allow him to put a sign in the front yard of the house advertising the painter’s new business. The investor chose a color and told the painter, “I don’t know anything about painting, so just do the job in your usual manner.” The investor paid the painter in advance for his services. As the painter was painting the side of the house, he accidentally flung a bucket of paint from the ladder where he was working. The bucket hit the investor’s neighbor in the head, causing the neighbor to suffer a concussion. The neighbor has asserted claims against the investor and the painter for his injuries.

If the neighbor recovers from the investor under a theory of vicarious liability, does the investor have any recourse against the painter?

A No, because payment by the investor was an acceptance of the work.
B No, because the investor selected the painter to do the work.
C Yes, because the judgment against the investor was based on vicarious liability.
D Yes, because the investor’s conduct was not a factual cause of the harm.

A

C Yes, because the judgment against the investor was based on vicarious liability.

How well did you know this?
1
Not at all
2
3
4
5
Perfectly
51
Q

A first-class passenger on an airplane flight from Atlanta to Chicago was seated next to a middle-aged salesman who was returning home after attending a business convention in Atlanta. The passenger, who was extremely exhausted after a long and hectic day, tried to sleep during the flight but was constantly being annoyed by the salesman. Once the flight departed, the salesman started ordering drinks of Scotch and water from the flight attendant, and became inebriated. When the passenger refused to talk to him, the salesman became very abusive to the passenger. Although there were many empty seats available on the plane in the first-class section, the passenger decided to remain seated next to the salesman. Finally, after the flight attendant had served the salesman his tenth drink of Scotch, the salesman became belligerent and punched the passenger in the mouth. The passenger’s two front teeth were knocked loose and she suffered a cut lip.

If the passenger asserts a claim against the airline based on negligence, what is the most likely outcome?

A. The passenger will prevail, because as a first class passenger, she was owed a special duty of care by the airline.

B. The passenger will prevail, because the flight attendant should have been aware that her conduct caused an unjustifiable risk of harm.

C. The passenger will not prevail, because the airline is not vicariously liable for the tortious conduct of its passengers.

D. The passenger will not prevail, because the passenger assumed the risk by not moving to another seat away from the salesman.

A

B. The passenger will prevail, because the flight attendant should have been aware that her conduct caused an unjustifiable risk of harm.

How well did you know this?
1
Not at all
2
3
4
5
Perfectly
52
Q

A city was experiencing a severe water shortage. To mitigate the problem, the city enacted an ordinance
limiting total water use and prohibiting the use of water by private citizens for any purpose other than
that of personal hygiene, cooking, and personal consumption. Use of water for landscaping, gardening,
washing cars, and the like was strictly prohibited and punishable by fines of up to $5,000.
A homeowner had a thriving vegetable garden which required daily watering. Because she limited her
personal water use, she felt justified in limited use of sprinklers to water her garden. The sprinklers were
timed to water the garden between 6:00 and 6:20 p.m. every night, before the homeowner got home
from work. As her garden was in her backyard, the homeowner was relatively confident that her water
use would go unnoticed by town officials, and it did. However, the sprinklers did not escape the attention
of a boy and his friend, the seven- and eight-year-old boys who lived on either side of the homeowner.
Noting that the homeowner’s sprinklers went off like clockwork every night at 6:00 p.m., just when temperatures
in the desert town were peaking, the boy and his friend made a habit of putting on their bathing
suits and sneaking into the homeowner’s backyard to run through the sprinklers, unbeknownst to their
mothers and to the homeowner.
One evening, in his enthusiasm, one of the boys tripped over the sprinkler spigot and scratched his
ankle. The wound went unnoticed by his mother until several days later, when she saw that her son was
limping. She took him to the doctor only to learn that he had developed a staph infection and required immediate
hospitalization. The boy was hospitalized for over a month before he recovered sufficiently to be
released, but his infected leg remained weak and his medical bills were extensive. The boy’s mother filed
suit against the homeowner to recover her medical expenses and lost wages, citing the homeowner’s
violation of the ordinance. The court accepted the statute as written.

Which of the following statements is most accurate?

(C) The boy’s mother will not prevail, because she was negligent in failing to control her child.

(B) The boy’s mother will prevail, because the attractive nuisance doctrine applies.

(D) The boy’s mother will not prevail, because the homeowner did not breach a duty of care or commit negligence per see.

(A) The boy’s mother will prevail, because the homeowner’s violation of the ordinance constituted negligence per se.

A

(D) The boy’s mother will not prevail, because the homeowner did not breach a duty of care or commit negligence per see.

How well did you know this?
1
Not at all
2
3
4
5
Perfectly
53
Q

The defendant suffers from a mental disability that diminishes his capacity to make rational decisions. The defendant attempted to assist the plaintiff in an emergency situation, and as a result of the defendant’s actions, the plaintiff suffered additional injuries.

To what standard of care will the defendant be held?

A. The standard of care of a reasonably prudent person.

B. The standard of care of a reasonably prudent person with diminished mental capacity.

C. The standard of care of a reasonably prudent person acting in the emergency situation.

D. The standard of care of a reasonably prudent person with diminished mental capacity acting in the emergency situation.

A

C. The standard of care of a reasonably prudent person acting in an emergency situation.

Discussion of correct answer: For purposes of analyzing a negligence action, the conduct of defendants whose cognitive abilities are diminished due to mental illness, mental disability, or intoxication is assessed without such diminishment of abilities. Therefore, the defendant, in this case, will not be held to the standard of care of a reasonably prudent person with diminished mental capacity. However, in an emergency situation, the defendant must act as a reasonable person would behave in the emergency situation.

54
Q

The plaintiff has been seeking medical care from his oncologist, the defendant. Recently, the defendant misdiagnosed the level of the plaintiff’s cancer, which impacted the course of treatment prescribed for the plaintiff. If the defendant would have properly diagnosed the plaintiff, the plaintiff would have a 45% chance of survival. However, due to the defendant’s malpractice, the plaintiff’s cancer is incurable and the prognosis is that that plaintiff will only live for one more month.

Under the traditional cause-in-fact rule, will the plaintiff succeed in a malpractice action against the defendant?

A. Yes, because the defendant was negligent in properly diagnosing the plaintiff’s condition.

B. Yes, because the defendant’s misdiagnosis decreased the plaintiff’s odds of survival.

C. No, because the plaintiff cannot prove that, but for the defendant’s malpractice, he would have survived.

D. No, because even though the plaintiff can successfully establish the prima facie elements of negligence, he cannot prove cause-in-fact by a preponderance of the evidence.

A

C. No, because the plaintiff cannot prove that, but for the defendant’s malpractice, he would have survived.

Discussion of correct answer: Cause-in-fact is one of the elements of a prima facie case of negligence, including malpractice claims. Under the traditional cause-in-fact rule, the plaintiff cannot succeed unless he can establish causation by more than 50%. Specifically, in a medical malpractice case, the plaintiff must show that it is more likely that not that, but for the defendant’s malpractice, he would have survived. Since the plaintiff in this case would have only had a 45% chance for survival if the defendant had not committed malpractice, the plaintiff will not be able to prevail.

55
Q

The plaintiff suffers from brittle bone disease. Because he is not paying attention to where he is going, the defendant bumps into the plaintiff while walking down the street. As a result, the plaintiff falls and fractures several bones, requiring hospitalization. If the plaintiff did not suffer from brittle bone disease, she would not have fractured any bones.

Is the defendant liable to the plaintiff for her injuries under a theory of negligence?

A. Yes, because the defendant will be liable for the plaintiff’s injuries even if the consequences are more severe than they would have been in a person not suffering from brittle bone disease.

B. Yes, because the defendant should have known that the plaintiff had from a condition that would cause her to suffer more severe injuries than a person not suffering from brittle bone disease.

C. No, because a defendant will not be liable for the plaintiff’s injuries if the consequences are more severe than they would have been in a person not suffering from brittle bone disease.

D. No, because the defendant did not have knowledge that the plaintiff had from a condition that would cause her to suffer more severe injuries than a person not suffering from brittle bone disease.

A

A. Yes, because the defendant will be liable for the plaintiff’s injuries even if the consequences are more severe than they would have been in a person not suffering from brittle bone disease.

56
Q

A woman was jogging on a cold day when she tripped and fell. As a result of the fall, she suffered a concussion and was unable to get up and seek medical attention. A man, who was training for a marathon, jogged past the woman. Unwilling to pause in his training, the man passed the woman without offering her any medical assistance, although he was in a position to easily help her. As a result, the woman suffered hypothermia from laying out in the cold.

Does the woman have a cause of action based in negligence against the man?

A. Yes, because the man had a duty to act because he could have helped the woman without causing any harm to himself.

B. Yes, because the man’s failure to act is considered a negligent omission.

C. No, because the man had no affirmative duty to take action to aid or protect the woman.

D. No, because the man could have placed himself in danger in trying to aid the woman.

A

C. No, because the man had no affirmative duty to take action to aid or protect the woman.

57
Q

An amateur inventor purchased a laser made for home hobbyist use and set it up on his roof to try to bounce a signal off of a satellite. He attempted to boost its power output with an over-the-counter voltage booster. However, because of a defective voltage regulation device in the laser that was present when it left the factory, the voltage booster caused the laser to overheat and explode. A fragment of molten quartz struck a neighbor, injuring him.

The neighbor brought an action against the laser manufacturer for the physical injuries he suffered as a result of the explosion. At trial, the neighbor presents evidence of the above facts. The manufacturer establishes that its laser was never intended to be used at the power levels necessary to reach a satellite, and that it exercised due care in the manufacture of the laser. The manufacturer moves for a directed verdict after the close of evidence.

How should the court rule?

A. Grant the motion, because its laser was never intended to be used at the power levels necessary to reach a satellite.

B. Grant the motion, because it exercised due care in the manufacture of the laser.

C. Deny the motion, because the jury must determine whether the neighbor is a foreseeable plaintiff.

D. Deny the motion, because the jury must determine whether the inventor’s use of the laser was foreseeable.

A

D. Deny the motion, because the jury must determine whether the inventor’s use of the laser was foreseeable.

The court should deny the motion because the jury must determine whether the inventor’s use of the laser was a reasonably foreseeable misuse. The manufacturer of an unreasonably dangerous defective product is strictly liable to foreseeable plaintiffs injured by the product. To establish a prima facie case in products liability based on strict liability in tort, the following elements must be proved:

(i) the defendant is a commercial supplier;
(ii) the defendant produced or sold a defective product;
(iii) the defective product was the actual and proximate cause of the plaintiff’s injury; and
(iv) the plaintiff suffered damage to person or property.

All of those elements are satisfied here. The manufacturer was a commercial supplier of the lasers. To hold the commercial supplier strictly liable for a product defect, the product must be expected to, and must in fact, reach the user or consumer without substantial change in the condition in which it is supplied. The facts establish that the defective voltage regulation device was present in the laser when it left the manufacturer’s production facilities. Courts extend the protection from defective products to all foreseeable plaintiffs, including members of the buyer’s family, guests, friends, and employees of the buyer, and bystanders, such as the neighbor here. While the neighbor has presented evidence of the prima facie case, the manufacturer has established that its laser was not intended to be used in the manner it was. Nevertheless, courts have required suppliers to anticipate reasonably foreseeable uses even if they are “misuses” of the product. Thus, misuse of the product will not preclude a products liability claim by the neighbor if the misuse was reasonably foreseeable, which is a determination to be made by the jury.

58
Q

A historical preservation group hired a restoration company to repair the crumbling facade of the town’s most treasured mansion. The restoration company constructed scaffolding that reached to the roof of the three-story building, covered the entire front of the mansion, and extended a short way along the two sides. The restoration company did not place a fence around the mansion or erect any other barriers or warning signs because it expected to complete the restoration within several days. The old mansion was located downtown, and children from a nearby neighborhood immediately began playing on and underneath the scaffolding in the evenings when no workers were present. Some of the older children moved one of the boards forming the walkway of the second level of scaffolding so that a portion extended out into the air. They used this “diving board” to leap across about four feet of intervening space and onto the roof of an adjacent building.

On the evening before the scaffolding was to be taken down as the project had been completed that day, a 14-year-old boy was taunted by his friends as being a “little baby” because he refused to jump off of the extended board and onto the adjacent building. Terrified of the height but unwilling to be branded a coward, the boy climbed to the second level and ventured out onto the board. Looking down, the boy’s fear of heights overcame him and he tried to retreat but fell, catching the edge of the board and hanging by his arms. His screams for help were heard by a passerby, who quickly climbed the scaffolding and crept out onto the “diving board,” reaching to grab the boy’s arms and pull him to safety.

Unfortunately, the passerby’s weight overbalanced the board, which was not secured in any way, and it toppled downward, throwing thepasserby and the boy to the ground below. The heavy board fell onto a six-year-old girl who was playing in a sand pile on the construction site. The girl was severely injured and her parents sued the restoration company.

What is the most likely result?

A. The restoration company is not liable, because the girl was a trespasser.

B. The restoration company is not liable, because the passerby was negligent in not realizing that the board would tip over due to his additional weight.

C. The restoration company is liable, because the danger to children from objects falling from the scaffolding was foreseeable.

D. The restoration company is strictly liable for the dangerous condition it created.

A

C. The restoration company is liable, because the danger to children from objects falling from the scaffolding was foreseeable.

Think Like a Lawyer

Land-occupiers having an “attractive nuisance” must take care to protect the area from child trespassers.

Step by Step Walkthrough

Step 1: A land occupier owes a duty of reasonable care to trespassing children with regard to artificial conditions on his premises if the following criteria are satisfied: (1) the land occupier must know or have reason to know that children are likely to trespass; (2) the land occupier must know or have reason to know that conditions on his property pose a serious risk of injury to children; (3) the children, because of their immaturity, must be unable to recognize or appreciate the risk; and (4) the burden to eliminate the risk must be small compared to the risk to the children.

Step 2: Here, the restoration company was aware of the presence of children trespassing. The company knew of the risk of serious injury. The children were clearly unable to appreciate the risk. The burden to secure the site was small, in comparison to the possible harm.

Step 3: Select the answer stating that the restoration company is liable, because the danger to children from objects falling from the scaffolding w

59
Q

A stockbroker became despondent because the economy was terrible and all of his clients had lost substantial amounts of money on their investments. The stockbroker decided to end his life by jumping off a bridge. He walked to the middle of the bridge, climbed over the railing, stood on the tiny ledge on the edge of the bridge, and prepared to jump. A taxi driver who happened to be crossing the bridge at that moment saw what the stockbroker was doing. The taxi driver slammed on his brakes, jumped out of his taxi, started running toward the stockbroker, and yelled, “Don’t do it!” Pleased to hear that someone cared about him, the stockbroker changed his mind about committing suicide and decided that he wanted to live. At that moment, however, the stockbroker started to lose his balance, and his feet began to slip off the small, grimy ledge of the bridge. Just as the stockbroker started to fall, the taxi driver reached him and grabbed the back of his jacket. The taxi driver hung on tightly to the jacket and nearly was pulled over the side of the bridge, but he managed to hang on.

Using every bit of strength he could muster, the taxi driver pulled the stockbroker back up and onto the bridge. The stockbroker was relieved to have been saved from falling. Unfortunately, the taxi driver suffered a severe injury to his back from the strain of hanging on to the stockbroker and pulling him back up onto the bridge. A statute in the jurisdiction makes attempted suicide a crime.

If the taxi driver asserts a claim against the stockbroker to recover damages for his injuries, will he prevail?

A. No, because the stockbroker did not intend to harm anyone other than himself.

B. No, because attempted suicide was a criminal offense in the jurisdiction.

C. Yes, because the stockbroker put himself in a position of peril.

D. Yes, because he succeeded in saving the stockbroker’s life.

A

C. Yes, because the stockbroker put himself in a position of peril.

60
Q

A man invited several friends to come over to his house to watch a movie on television. While they were watching the movie, freezing rain fell outside, coating everything with a thin layer of very slippery ice. When the movie ended, one of the man’s guests slipped on the ice as soon as he took a step out of the door and onto the front stoop of the man’s house.

Which of the following would best describe the duty of care owed by the man to his guest?

A. No duty of care.

B. A duty to inspect the premises for unknown dangers and disclose their existence to others.

C. A duty to warn of any known dangerous condition on the premises.

D. An absolute duty of care.

A

C. A duty to warn of any known dangerous condition on the premises.

61
Q

A jogger came across a bicyclist who had fallen from his bicycle and broken his leg. The jogger told the bicyclist that she would go and get help for him. However, the jogger failed to seek help on behalf of the bicyclist. Several minutes later, a man spotted the bicyclist and offered to help him. The bicyclist denied the man’s offer because he believed that the jogger had already gone for help. No one else came to help the bicyclist and he was not found for several days, by which time he had suffered severe dehydration.

Under the majority rule, is the jogger liable for the injuries suffered by the bicyclist after she failed to assist him?

A. Yes, because the jogger had a duty to exercise reasonable care in rendering aid to the bicyclist.

B. Yes, because the bicyclist relied on the jogger’s promise to his detriment.

C. No, because the jogger gratuitously promised to take action to aid the bicyclist.

D. No, because a rescuer never has a duty towards the party being rescued.

A

C. No, because the jogger gratuitously promised to take action to aid the bicyclist.

Discussion of correct answer: Under the majority rule, a defendant who gratuitously promises to take action to aid a plaintiff has no duty to actually take the promised action, even if the defendant has relied on the promise to his detriment. Here, the jogger gratuitously promised to help the bicyclist. By doing so, the jogger did not create any sort of duty to aid the bicyclist. This is true even though the bicyclist denied a subsequent offer for help to his detriment. Note that if a defendant gratuitously promises to aid a plaintiff, once the defendant attempts to give the promised aid, the defendant has a duty to exercise reasonable care in doing so. However, in this case, this rule does not apply because the jogger made no attempt to aid the bicyclist.

62
Q

A mother bought her 14-year-old son a new BB gun for his birthday. The next day the son decided to shoot at cars passing by the house. From his bedroom window, the son would aim at the hubcaps of the cars. The son saw a car slowly coming down the street that was driven by an elderly woman. The son aimed for the hubcaps but instead shot through the passenger side window. While the elderly woman was not hit, the noise of the glass breaking caused her to lose control of the car and crash into a telephone pole. The elderly woman brought a suit for damages against the mother.

Will the elderly woman likely prevail in her suit for damages?

A. Yes, because a parent is vicariously liable for the negligence of a child.

B. Yes, because a parent may be liable for the intentional torts of a child.

C. No, because a parent is not vicariously liable for the negligence of a child.

D. No, because a parent is not liable for the intentional torts of a child.

A

B. Yes, because a parent may be liable for the intentional torts of a child.

Discussion of correct answer: A parent has a duty to exercise reasonable care to control a child to prevent the child from intentionally harming another person or from creating an unreasonable risk of harm to another person when the parent knows or has reason to know she has the ability to control the child and knows or should know of the necessity and opportunity to exercise control over the child. Under this standard, the mother may be liable for the son’s intentional tort. Here, the facts indicate that the mother may face liability if the elderly woman can prove the mother knew or had reason to know of her ability to control the son and the necessity to control the son.

Think Like a Lawyer

If your client buys his minor child a gun, the client needs to control the child so that third parties are not harmed.

Step by Step Walkthrough

Step 1: A parent is under a duty to exercise reasonable care to control his minor child so as to prevent the child from intentionally harming others or creating an unreasonable risk of bodily harm to them, if the parent: (1) knows or has reason to know that he has the ability to control his child; and (2) knows or should know of the necessity and opportunity for exercising such control.

Step 2: Here, the mother bought her minor child a gun. The mother should have had reason to know that she had the ability to control the child and the necessity for exercising that control. This is so because the child is going to start pointing the gun at things and shooting them. However, it is clear from the facts that the mother did not control the child. Shooting a gun at people and hitting them, or hitting things near them, equals the intentional tort of battery. The mother will end up being liable for the battery.

Step 3: The elderly woman will prevail in her suit for damages, because a parent may be liable for the intentional torts of a child. Select this answer.

Step 4: Ignore the answer choice regarding the negligence of a child, because this child was shooting at cars on purpose. This shows battery, not negligence.

Step 5: Ignore the two answer choices stating that the elderly woman will not prevail. Negligence by the child, as already discussed, is not at issue in this case. Intentional torts can indeed form the basis for parental liability, if the parent knew he could control the child and knew the need for controlling the child, as is the case here.

63
Q

A pilot decided to fly his small airplane to another city for a weekend visit. He invited one of his best friends to join him as a passenger on the trip. Near the end of the flight, the pilot saw a cloud of fog ahead but decided that he could navigate through it. He got lost in the fog and crashed the plane into the side of a mountain. The pilot and his passenger were both killed instantly in the crash.

If the passenger’s family brings a wrongful death action against the pilot’s estate, what is the most likely outcome?

A. The plaintiffs will recover under strict liability, because the pilot engaged in an abnormally dangerous activity.

B. The plaintiffs will recover under negligence, because the pilot failed to exercise reasonable care in the operation of his airplane.

C. The plaintiffs will not recover, because the passenger did not pay to go on the flight.

D. The plaintiffs will not recover, because the passenger unreasonably exposed himself to a known danger.

A

B. The plaintiffs will recover under negligence, because the pilot failed to exercise reasonable care in the operation of his airplane.

64
Q

A man negligently set fire to apartment building. The fire caused a panic and residents rushed to try to escape the burning building. In the rush to get out, a woman was knocked to the ground and trampled by the other residents. The woman later sued the man for her injuries.

Which of the following statements is most accurate?

(B) The woman will recover, because the man is responsible for the foreseeable injuries sustained by the woman.

(D) The woman will not recover, because her injuries were sustained by an unforeseeable force.

(A) The woman will recover, because the man is responsible for all injuries sustained by the woman.

(C) The woman will not recover, because her injuries were sustained in an unforeseeable manner.

A

(B) The woman will recover, because the man is responsible for the foreseeable injuries sustained by the woman.

65
Q

A deaf bicyclist was riding on a trail commonly shared by bicyclists and pedestrians. The bicyclist hit and injured a pedestrian, who tried to warn the bicyclist that he was about to hit him by yelling, “Stop!”

In a negligence action by the pedestrian against the bicyclist, to what standard of care will the bicyclist be held?

A. He will be held to the same standard of care as a reasonably prudent deaf person.

B. He will be held to the same standard of care as a reasonably prudent person.

C. He will be held to the same standard of care as any person who is also deaf.

D. There is not enough information from the facts to make this determination.

A

A. He will be held to the same standard of care as a reasonably prudent deaf person.

66
Q

A supermarket had just reopened after a six-month renovation period. The renovations included the repair, replastering, and repainting of the entire ceiling and walls by the painting company. The day following the reopening of the supermarket, the store’s manager noticed small fragments of plaster on the floor, which appeared to have fallen from a part of the ceiling about 10-square-feet in area. The manager immediately posted signs in that area that read: “Caution: Falling Plaster.” In addition, the manager promptly called the painting company and requested a repairman to attend to the problem. That afternoon, a shopper was shopping in the supermarket and wanted to purchase some canned goods in the posted area. Moving quickly, the shopper reached for the desired items. At that very moment, a section of the ceiling fell and struck the shopper, injuring her very seriously.

If the shopper asserts a claim against the supermarket, what is the likely outcome?

A. The shopper will prevail, because the supermarket failed to take adequate precaution to ensure the safety of its patrons against such an unreasonable risk of harm.

B. The shopper will prevail, because the supermarket is vicariously liable for the negligence of its independent contractor in the performance of a non-delegable duty.

C. The shopper will not prevail, because the injury was the fault of the painting company.

D. The shopper will not prevail, because the supermarket posted signs warning customers of the risk of harm.

A

A. The shopper will prevail, because the supermarket failed to take adequate precaution to ensure the safety of its patrons against such an unreasonable risk of harm.

A possessor of land is required to exercise reasonable care to protect invitees, such as customers in a store, from harm. This is a nondelegable duty, meaning that the possessor will be obligated to exercise reasonable care, even where the danger is created by the acts of an independent contractor doing work on the property. Although giving a warning about a danger may be sufficient in some circumstances, reasonable care may require the possessor of the land to do more than just post a warning sign. Here, although the manager posted a warning sign in the potentially dangerous area of the supermarket, more adequate precautions should have been taken under the circumstances. Consequently, the shopper would be entitled to recover damages in negligence against the supermarket.

67
Q

A hobbyist went out fishing on a lake at dawn. Few other people were on the lake so early in the morning, and the hobbyist enjoyed the peace and quiet. A short while later, the hobbyist noticed a canoe with one occupant heading in his direction. The canoe stopped about 50 yards away. The hobbyist was glad that the canoe didn’t come any closer, because he didn’t want the movement of the canoe to disturb the fish. The hobbyist then watched as the canoeist stood up in the canoe and began moving around, apparently attempting to organize his fishing tackle.

Suddenly, the canoeist’s canoe overturned. The canoeist, who was not wearing a life preserver, was thrown into the water. At that same moment, the hobbyist felt a strong tug on his fishing line. He didn’t want to lose the fish, so he decided to reel in the fish before going to see if the canoeist needed any assistance. The hobbyist figured that, at worst, the canoeist would have to tread water for a few minutes until the hobbyist got to him. Unfortunately, canoeist did not know how to swim, and by the time the hobbyist made his way over to the canoeist and pulled him out of the water, the canoeist was no longer breathing. Although the hobbyist promptly administered CPR and the canoeist was eventually revived by paramedics, he suffered severe brain damage.

If the canoeist’s parents sue the hobbyist for the canoeist’s injuries, what is the hobbyist’s best argument in support of a motion to dismiss their claim?

A The hobbyist’s delay in assisting the canoeist was reasonable.
B The hobbyist had no duty to help the canoeist.
C The canoeist was contributorily negligent.
D The hobbyist properly administered CPR to the canoeist so that the canoeist could be revived by the paramedics.

A

B The hobbyist had no duty to help the canoeist.

68
Q

Driving home from work one evening, a man suddenly had engine trouble. Fearing that he would permanently damage his engine, the man immediately pulled to the side of the road and parked. The man failed to notice he had parked five feet from a fire hydrant. The man got out of his car to call for help. Shortly thereafter, a car negligently crashed into the man’s parked car. A local city ordinance made it illegal to park within 10 feet of a fire hydrant. The man sued the other driver for the damage to his car.

What result is most likely?

A. Judgment for the other driver, because the man’s violation of the city ordinance constituted comparative negligence per se.

B. Judgment for the other driver, because the man’s parking his car next to the fire hydrant was the cause in fact of the accident.

C. Judgment for the man, because the city ordinance was designed to provide the fire department access to the fire hydrant.

D. Judgment for the man, because the other driver had the last clear chance to avoid the accident.

A

C. Judgment for the man, because the city ordinance was designed to provide the fire department access to the fire hydrant.

69
Q

A man invited several friends to come over to his house to watch a movie on television. While they were watching the movie, freezing rain fell outside, coating everything with a thin layer of very slippery ice. When the movie ended, one of the man’s guests slipped on the ice as soon as he took a step out of the door and onto the front stoop of the man’s house.

Which of the following would best describe the duty of care owed by the man to his guest?

(B) A duty to inspect the premises for unknown dangers and disclose their existence to others.

(D) An absolute duty of care.

(A) No duty of care.

(C) A duty to warn of any known dangerous condition on the premises.

A

(C) A duty to warn of any known dangerous condition on the premises.

70
Q

After numerous traffic accidents at a certain intersection, a city decided to install a traffic signal there. The number of traffic accidents decreased dramatically. Six months later a concerned citizen, who lived near the intersection, noticed that the traffic light had stopped working. He immediately reported the outage to the city. Three days later, a driver came to a stop and then carefully proceeded through the intersection. However, he was hit by another car driven by a grandfather, who had failed to notice the signal was not working. The driver sued the city.

Which of the following is the driver’s strongest argument?

A. The city failed to exercise due care by letting at least three days pass without repairing the signal.

B. The driver was behaving reasonably when he entered the intersection since he stopped and then carefully proceeded.

C. The combination of the city’s failure to repair the signal and the grandfather’s failure to notice the signal was out was the cause of the collision.

D. A municipality is strictly liable for any injuries arising from its failure to properly control traffic flow.

A

A. The city failed to exercise due care by letting at least three days pass without repairing the signal.

71
Q

A construction company was doing repairs and replacing portions of a sidewalk and railing next to a lake. The construction crew started tearing out the old sidewalk and railing, but stopped work when it started to get dark. The construction crew left without putting up a warning sign or barrier around the work area. A few hours later, a jogger came along the sidewalk. Not realizing the construction work was in progress there, the jogger stumbled and fell at the spot where the construction crew had torn up the sidewalk and railing. The jogger fell into the lake.

As the jogger was attempting to stay afloat, he began screaming, “Help! Help! I can’t swim. I’m drowning.” His screams attracted the attention of a person who was passing on his bicycle. The cyclist immediately hurried to assist the jogger. As the cyclist was leaning over the edge of the lake, trying to help the jogger get out of the water, he lost his balance and fell into the lake. Both the jogger and cyclist suffered serious bodily injuries before they were pulled out of the water by police.

In a negligence action by the cyclist to recover for his personal injuries, what is the probable outcome?

A. The construction company will be held liable, because the cyclist’s attempt to rescue the jogger was foreseeable.

B. The construction company will be held liable, because the construction company would be strictly liable to anyone injured by the failure to put adequate warnings or barriers around the site of the sidewalk repairs.

C. The construction company will not be held liable, because the cyclist assumed the risk by leaning over the edge of the lake.

D. The construction company will not be held liable, because the construction company could not foresee that anyone would be hurt while trying to rescue someone from the lake.

A

A. The construction company will be held liable, because the cyclist’s attempt to rescue the jogger was foreseeable.

72
Q

One morning, a small, frail, elderly man was waiting at a crowded bus stop for the city bus to go to the local senior center. When the bus arrived, a shopper, who was carrying a large handbag, got on in front of him. As the man struggled to manage the steep stairs onto the bus, the shopper’s handbag, which was quite heavy, swung back and hit him, throwing him off balance. The man fell back into the street and broke his hip. It was later revealed that the shopper had been carrying a large handgun in her purse, which was what had made the bag so heavy. While the shopper had a permit for the weapon, the jurisdiction in question had enacted an ordinance prohibiting the carrying of concealed weapons.

The man files a lawsuit against the shopper to recover his medical expenses on grounds of negligence per se.

A. No, because the harm was not of the type that the statute was designed to prevent.

B. No, because the man’s injuries were unforeseeably extensive.

C. Yes, because the handbag was heavy due to the gun, and the man was within the class of people the statute was designed to prevent.

D. Yes, because the man can prove causation and damages.

A

A. No, because the harm was not of the type that the statute was designed to prevent.

73
Q

A woman and a man, who worked for the same company, regularly commuted to work together. Each drove on alternate days. One day when the woman was to drive, she arrived at the man’s house at the appointed time. Shortly after they left, the woman informed him that she had received a notice in the mail the day before from the manufacturer of her car warning her that, due to the manufacturer’s negligence, there was a defect in her car’s brake system and she should not drive the car until the brakes were repaired. The man stated they should return to his house and take his car, but the woman replied that there was a dealership next to their place of work and she would have the brakes repaired during the day. She further stated that she was sure the brakes would hold up on the trip to work. Shortly thereafter, the traffic in front of the woman came to a stop. When she applied the brakes they failed, causing the woman’s car to strike the stopped car in front of her. The man was injured in the accident.

If the man brings a negligence action against the manufacturer, which of the following most accurately describes the significance of the woman’s negligence to the litigation?

A. The woman’s negligence was the proximate cause of the man’s injuries.

B. The woman’s negligence was a foreseeable danger the risk of which was increased by the manufacturer’s negligence.

C. The woman’s negligence was an independent, intervening force which superseded the manufacturer’s liability.

D. The woman’s negligence was the legal cause of the man’s injuries.

A

A. The woman’s negligence was the proximate cause of the man’s injuries.

Think Like a Lawyer

A superseding cause must always be unforeseeable in order to cut off liability for the defendant.

Step by Step Walkthrough

Step 1: Negligence requires: (1) duty; (2) breach of that duty; (3) causation, both actual and proximate; and (4) damages. Actual cause means the cause-in-fact of the injury. Legal cause, also known as proximate cause, means whether the defendant owed a duty of reasonable care to the plaintiff as one who could foreseeably be injured. A defendant owes a duty of reasonable care only to foreseeable plaintiffs, to act as the reasonable prudent person would under the same circumstances.

Step 2: A superseding cause must be unforeseeable in order to cut off liability.

Step 3: Here, the woman breached her duty of care to the man by picking him up in a car that had bad brakes. Consequently, her negligence was a factor in the accident. However, the man is suing the manufacturer, who was strictly liable for the parts on the car. The manufacturer’s liability was not cut off by the woman’s poor judgment, which was foreseeable. Each car owner that received the recall notice was going to drive their car to a repair station. Thus, the possibility of accidents on the way to having the car repaired was quite foreseeable. Select the answer stating that the woman’s negligence was a foreseeable danger, the risk of which was increased by the manufacturer’s negligence.

Step 4: Eliminate the answer choice saying that the woman’s negligence was the proximate cause of the man’s injuries, because this is only partly true. This answer choice neglects to discuss the relationship to the manufacturer.

Step 5: Eliminate the answer choice saying that the woman’s negligence was an independent, intervening force which superseded the manufacturer’s liability, because this is not true. A superseding cause must always be unforeseeable.

Step 6: Finally, eliminate the answer choice saying that the woman’s negligence was the legal cause of the man’s injuries, because this is only partly true. This answer choice neglects to discuss the relationship to the manufacturer, whose actions were also a legal cause of the man’s injuries.

74
Q

A property owner in a residential area owns a home on several acres on which he keeps a horse. The horse roams free and grazes on the property. There is no fence bordering the property. A seven-year-old neighborhood child entered the property to pet the horse. When the child approached the horse from behind, he was kicked and sustained injuries to his chest and ribs.

In a negligence action for damages against the property owner, will the child likely prevail?

A. Yes, because while the child was a trespasser, the property owner owed the child a heightened duty of care under the attractive nuisance doctrine

B. Yes, because the child was an invitee.

C. No, because the child was a trespasser.

D. No, because the child’s negligence in approaching the horse was the proximate cause of the injury.

A

A Yes, because while the child was a trespasser, the property owner owed the child a heightened duty of care under the attractive nuisance doctrine

75
Q

A man negligently hit a woman with his car. The woman was rushed to the hospital and sent into surgery. During the course of the operation, the doctor was careless and caused the woman more injuries.

If the woman sued the man, which of the following statements is most accurate?

A The woman should recover for all of her injuries, because they were foreseeably caused by the man.
B The woman should recover for all of her injuries, because the thin-skulled plaintiff rule provides that a defendant takes their victim as they find them.
C The woman should only recover for those injuries sustained in the accident, and she must sue doctor as a separate tortfeasor for the damages sustained during surgery.
D The woman should only recover for those injuries sustained in the accident, because it is not foreseeable that a doctor would be negligent.

A

A The woman should recover for all of her injuries, because they were foreseeably caused by the man.

76
Q

A police officer was patrolling a particular neighborhood as part of a police department’s “Safe Block” initiative. The police officer’s duties included checking vacant apartment buildings and houses. After giving notice to the owner a couple of days beforehand, between 9:00 p.m. and 10:00 p.m. one evening, the police officer approached a vacant apartment building to verify that the basement door was locked. As he slowly descended the stairs that led from the sidewalk down to the basement door, grasping the handrail and shining his flashlight, the cement that was covering the steps broke, causing him injuries. The property owner’s maintenance staff had previously informed the owner that the steps would need repair. The property owner moved for summary judgment, arguing that he did not owe the police officer a duty. The trial court agreed and granted summary judgment, concluding that defendant did not owe the police officer any duty. The police officer has filed an appeal.

Should the appellate court reverse the trial court’s ruling?

A. Yes, because the police officer was an invitee.

B. Yes, because the police officer was privileged to enter the property.

C. No, because the property owner did not owe the police officer a duty of care because it was a vacant building.

D. No, because the police officer’s negligence was a superseding intervening cause.

A

B. Yes, because the police officer was privileged to enter the property.

77
Q

A pedestrian was walking in front of a hotel in the downtown area when a chair was thrown from an unidentified window. The chair struck the pedestrian on the head, knocking her unconscious. When the pedestrian recovered consciousness, she found herself in a nearby hospital with a doctor in attendance. An examination revealed that the pedestrian had suffered a concussion and severe head lacerations. A subsequent investigation revealed that the chair had, in fact, been thrown from a window at the hotel by a hotel employee who was angry with a hotel guest who insulted the employee. The pedestrian asserted a claim against the hotel for battery.

Will the hotel prevail?

A. No, because the chair was within the control of the hotel.

B. No, because the hotel is vicariously liable for the intentional torts of its employees.

C. Yes, because the chair was not within the control of the hotel at the time the pedestrian was injured.

D. Yes, because the hotel is not vicariously liable for the intentional tort of this employee.

A

D. Yes, because the hotel is not vicariously liable for the intentional tort of this employee.

78
Q

A man and his friend enjoyed hunting together. One weekend, they went on a quail hunting expedition in a remote wilderness area deep in the mountains. They were hunting in a very desolate area surrounded by mountainous terrain located at least 50 miles from any habitation. With hunting rifles in hand, they were ambling around but no game was visible. After a rather uneventful morning with nothing to shoot at, the man suddenly spotted a large condor. The condor was an endangered species, and to shoot one was a criminal offense. Unable to resist the temptation, the man took a shot at the condor. The bullet missed the vulture, ricocheted off a tree, and struck a state forest ranger who was hiding in a secluded area, watching a trail frequented by drug smugglers. The bullet hit the forest ranger in the eye and permanently blinded him. Neither the man nor his friend was aware of the forest ranger’s presence.

If the forest ranger asserts a claim against the man to recover damages for his injury, will he prevail?

A. Yes, because the man’s shooting the gun was the actual cause of the forest ranger’s injury.

B. Yes, because the man intended to shoot the condor.

C. No, because the man had no reason to anticipate the presence of another person in such a remote area.

D. No, because the bullet ricocheted off a tree and hit the forest ranger.

A

C. No, because the man had no reason to anticipate the presence of another person in such a remote area.

Discussion of correct answer: A popular Multistate testing area deals with proximate or legal cause. In order to be liable for negligence, defendant’s conduct must constitute the legal or proximate cause of plaintiff’s harm or injury. According to the rule enunciated in the Palsgraf case, Judge Cardozo stated that a defendant’s duty of care is owed only to foreseeable plaintiffs (i.e., those individuals who are within the risk of harm created by defendant’s unreasonable conduct). Choice (C) is the best answer because it addresses the fact that the forest ranger was an unforeseeable plaintiff to whom no duty of care was owed. Choice (B) is incorrect because transferred intent does not apply chattel-to-person, only person-to-person.

79
Q

A concert pianist suffered a strained wrist, so he went to visit a doctor. The doctor explained to her that the fastest way to regain complete use of the wrist was to treat it with a series of injections of a drug that blocked pain and reduced inflammation. The concert pianist had discussed with the doctor her need to be ready for an important recital in three weeks. The drug was known to cause calcium buildup in joint tissue in approximately one to two percent of patients treated with it. For most individuals experiencing this side effect, the calcium buildup did not result in noticeable limitation of movement. If it did, the buildup could be surgically removed, and after a brief recovery period (one to two months), complete movement was restored.

The doctor did not inform the concert pianist of these facts, and she began a course of treatment consisting of three injections per week for three weeks. At the end of the second week, the concert pianist received in the mail the current edition of a popular health magazine. To her dismay, it contained an article describing the dangers of using the drug that the doctor recommended, pointing out that calcium buildup in the knuckles could be devastating to a pianist. The concert pianist discontinued treatment immediately and was not able to perform in her upcoming recital due to pain in her wrist, but not because of any calcium buildup in her knuckles. The concert pianist sought treatment from another physician, who recommended a two-month hiatus from piano playing followed by six months of physical therapy. After that time, the concert pianist was able to resume her playing.

Which of the following would be most relevant in determining whether the concert pianist has a valid cause of action against the doctor?

A Whether the concert pianist would not have consented to the use of the drug if she knew about the risks to her mobility.
B Whether a reasonable person would have considered the information about calcium buildup important.
C Whether the doctor had weighed the risks and benefits of using the drug before prescribing it.
D Whether the concert pianist suffered no harm from the use of the drug.

A

D Whether the concert pianist suffered no harm from the use of the drug.

80
Q

A homeowner purchased a new vacuum cleaner. A few days later, the homeowner received a severe electric shock while using the vacuum cleaner. The homeowner realized that there was a short in the wiring of the vacuum cleaner. The homeowner called the store that sold the vacuum cleaner and arranged to return it for a refund. The homeowner was busy, however, and would not have time to make a trip to the store for several days, so he put the vacuum cleaner in a corner in his living room to store it until he had a chance to take it to the store. The next day, the homeowner had an old friend from out of town arrive for a three-day visit. The homeowner had a spare room and loved to have overnight guests, so he had invited this old friend to come for a visit and was excited to see her. Shortly after the guest’s arrival, however, the homeowner had to go to work for a few hours, so he told his friend to make herself comfortable in the house while he went to his office. While the homeowner was away from the house, his friend got hungry and made herself a snack. After finishing the snack, she realized that she had dropped a lot of crumbs on the carpet. Wanting to be a polite guest, she decided to clean up the mess herself before the homeowner returned. She saw the vacuum cleaner in the corner, plugged it in at an electric outlet, and started to vacuum up the crumbs when she received a very strong electric shock as a result of the vacuum
cleaner’s faulty wiring.

If the guest asserts a claim against the homeowner, what is the most likely outcome?

(B) The guest will recover, because the homeowner had a duty to make a reasonable inspection of the home to discover any unknown dangers.

(D) The guest will not recover, under the family purpose doctrine.

(A) The guest will recover, because the homeowner knew about the hazardous condition of the vacuum cleaner and yet failed to warn the guest.

(C) The guest will not recover, because the homeowner did not create the risk of harm.

A

(A) The guest will recover, because the homeowner knew about the hazardous condition of the vacuum cleaner and yet failed to warn the guest.

81
Q

A teenage boy and his friend were playing paintball at the friend’s father’s farm. The paintballs were designed to break on impact, spraying paint but not physically injuring anyone. The boy pointed his
paintball gun at the friend and fired a paintball at him. The paintball missed the friend and instead hit a horse that was grazing nearby. The horse, startled and in pain, started bucking and kicked up a rock that hit the friend in the head, causing him to lose vision in his right eye.

In a negligence action against the boy, will the friend recover?

(B) No, because shooting the gun was not the actual cause of the friend’s injury.

(D) Yes, because the boy shot the gun at his friend, which is sufficient for recovery under transferred intent.

(C) Yes, because paintball is an adult activity.

(A) No, because shooting the gun was not the proximate cause of the friend’s injury.

A

(A) No, because shooting the gun was not the proximate cause of the friend’s injury.

82
Q

A plaintiff and defendant were playing football. The plaintiff knew that the defendant had a reputation for being an extremely competitive player. During the game, the plaintiff’s eye was severely injured when
the defendant’s finger came into contact with it while trying to catch a pass. The plaintiff sought medical attention, and the doctor diagnosed him with a scratched cornea. The plaintiff subsequently brought a
negligence action against the defendant.

Will the plaintiff prevail?

(B) No, because the plaintiff voluntarily played the game.

(C) Yes, because the defendant’s actions fell below the standard of care.

(A) No, because the plaintiff impliedly consented to this type of injury.

(D) Yes, because this type of physical injury is beyond the scope of consent.

A

(A) No, because the plaintiff impliedly consented to this type of injury.

83
Q

Two friends were playing basketball in the park. At one point, the basketball bounced off the backboard and rolled into the street that ran along the park. One of the friends ran after the basketball and ran into the street to retrieve it without first checking to see if cars were coming. The friend ran into the path of a car, whose driver failed to see the friend because the driver was dialing her cell phone at the time. The car hit the friend, causing the friend serious injuries. The friend brought an action against the driver for his injuries.

If it is determined that both the friend and the driver were negligent, what, if anything, will the friend recover?

A. Nothing, because the collision would never have occurred but for the friend’s negligence.

B. His total damages reduced by the percentage to which his negligence contributed to his own injuries, but if that percentage exceeds 50%, he will recover nothing.

C. His total damages reduced by the percentage to which his negligence contributed to his own injuries.

D. His total damages, because the driver had the last clear chance to avoid the collision.

A

C. His total damages reduced by the percentage to which his negligence contributed to his own injuries.

84
Q

A man bought a ready-cooked chicken and some bread at the supermarket. He then returned home, where he was joined by a friend for dinner. The friend was injured when she bit into the chicken and encountered a piece of broken glass. The friend sues the store.

Will she prevail?

A. Yes, because the store failed to exercise due care.

B. Yes, because the store is strictly liable.

C. No, because she did not purchase the chicken.

D. No, because the store only cooked the chicken.

A

B. Yes, because the store is strictly liable.

85
Q

A taxicab driver stopped at a convenience store to buy a snack. While he was handing his money to the store clerk at the cash register, a police officer entered the store. The taxi driver had received several traffic tickets from the police officer, and he felt that the police officer had been very rude to him. Seeking to gain some revenge, the taxi driver immediately began to taunt and berate the police officer, calling him a variety of profane and insulting names. The police officer was going through a series of personal difficulties and was feeling very emotionally fragile. As the taxi driver’s insults rained down on him, the police officer suddenly snapped. He whipped out his gun and fired a shot at the taxi driver. The bullet missed the taxi driver but hit the store clerk, who was standing behind the taxi driver. The clerk survived the shooting but suffered a gunshot wound to his shoulder.

In an action by the injured clerk against the taxi driver to recover for his personal injuries, what is the likely outcome?

A. The clerk will recover, because the taxi driver’s conduct was the proximate cause of the clerk’s injuries.

B. The clerk will recover, because it was foreseeable that the taxi driver’s conduct would result in the clerk being shot.

C. The clerk will not recover, because the shooting was not a foreseeable consequence of the taxi driver’s conduct.

D. The clerk will not recover, because the police officer intended to shoot the taxi driver, not the store clerk.

A

C. The clerk will not recover, because the shooting was not a foreseeable consequence of the taxi driver’s conduct.

86
Q

A patient went to a doctor for treatment of a painful ear condition, which had caused her to experience loss of balance, dizziness, and nausea. While the doctor was examining her inner ear, he noticed a lump of wax. As the doctor was removing the lump, he also noticed what appeared to be the rear legs of an insect. With utmost care, the doctor extracted the foreign object from the patient’s ear. The doctor then examined the substance, which proved to be a cockroach. As he held the insect in front of the patient to see, she became hysterical, fainted, and fell to the floor, breaking her collarbone. The incident caused the patient to develop a phobia and she became deathly afraid of insects.

In a suit for damages against the doctor, which of the following torts would provide the patient with her best theory of recovery?

A. Battery.

B. Negligence.

C. Negligent infliction of emotional distress.

D. Intentional infliction of emotional distress.

A

C. Negligent infliction of emotional distress.

87
Q

A plaintiff and defendant were playing basketball. The plaintiff knew that the defendant had a reputation for being an extremely competitive player. During the game, the plaintiff’s eye was severely injured when the defendant poked the plaintiff in her face while trying to get a rebound.

If the plaintiff brings a cause of action against the defendant, will the plaintiff prevail?

A. Yes, because she suffered a harmful or offensive contact.

B. Yes, because the defendant’s actions fell below the standard of care.

C. No, because the plaintiff assumed the risk of such injury.

D. No, because the defendant did not intend to injure the plaintiff.

A

C. No, because the plaintiff assumed the risk of such injury.

88
Q

A well-known Oscar-winning producer who owned the nation’s leading special effects studio employed a creative designer who was recognized throughout the industry as one of the top talents in the special effects field. A competitor of the producer who wanted to hire the designer told him that that the producer was in negotiations to sell his studio to a major Hollywood film company. In fact this statement was not true, and the competitor made the statement without any knowledge of its truth or falsity. The designer, whose employment contract with the producer was terminable at will by either party, agreed to be hired by the competitor. The producer was very upset when he learned of the competitor’s action because he knew that the designer would be almost impossible to replace. The producer brought suit against the competitor.

If the producer establishes the above facts and that he suffered damages, which of the following is likely to be true?

A. He will recover for intentional interference with business relations, because the competitor used improper means to hire the designer away from the producer.

B. He will recover for defamation, because the producer can show actual damages from the competitor’s statement.

C. He will not recover for intentional interference with business relations, because the designer was an at-will employee of the producer.

D. He will not recover for defamation, because the producer did not show that the competitor knew that his statement about the producer’s plans was false.

A

A. He will recover for intentional interference with business relations, because the competitor used improper means to hire the designer away from the producer.

The producer can recover for intentional interference with business relations because the competitor used improper means to hire the designer away. To establish a prima facie case for interference with contract or prospective economic advantage, the plaintiff must prove:

(i) the existence of a valid contractual relationship between the plaintiff and a third party or a valid business expectancy of the plaintiff;
(ii) the defendant’s knowledge of the expectancy;
(iii) intentional interference by the defendant that induces a breach or termination of the relationship or expectancy; and
(iv) damage to plaintiff.

While an interferer’s conduct may be privileged where it is a proper attempt to obtain business for the interferer, not only the ends but also the means of persuasion used must be proper. Here, the competitor made an intentional misrepresentation with the intent to induce the designer to leave the producer’s employment, and the producer has shown that he suffered actual damage from the designer’s departure.

89
Q

A pedestrian was walking in front of a hotel in the downtown area when a chair was thrown from an unidentified window. The chair struck the pedestrian on the head, knocking her unconscious. When the pedestrian recovered consciousness, she found herself in a nearby hospital with a doctor in attendance. An examination revealed that the pedestrian had suffered a concussion and severe head lacerations. A subsequent investigation revealed that the chair had, in fact, been thrown from a window at the hotel. None of the hotel’s employees or guests, however, admitted culpability for the incident.

If the pedestrian asserts a claim against the hotel for negligence, will the doctrine of res ipsa loquitur enable her to recover?

A. Yes, because the chair was within the control of the hotel.

B. Yes, because a chair is not usually thrown from a window in the absence of someone’s negligence.

C. No, because the chair was not within the control of the hotel at the time the pedestrian was injured.

D. No, because the hotel is not vicariously liable for the tortious conduct of its employees.

A

C. No, because the chair was not within the control of the hotel at the time the pedestrian was injured.

90
Q

A bottler markets water in lightweight plastic bottles that are sold by grocery stores, sporting goods stores, and other retail outlets. A hiker purchased several bottles of the water from a retailer and took them with him on a hike. While the hiker left his backpack unattended, a thief took one of the unopened containers without permission and drank some of the water. He immediately became violently ill. Tests were run on the water and showed that it contained impurities.

If the thief maintains a negligence action against the bottler, which of the following arguments would be the most helpful to the bottler in avoiding liability?

A. The retailer had ample opportunity to test and inspect samples of the bottled water for purity and failed to do so.

B. The bottler bottled its water in compliance with numerous statutes that regulate the process of bottling water for human consumption.

C. The thief has failed to introduce any evidence at trial as to how the impurities got into the water he drank, and therefore has not met his burden of proof.

D. No reasonable person would have foreseen that the water would have been stolen and consumed by a thief.

A

B. The bottler bottled its water in compliance with numerous statutes that regulate the process of bottling water for human consumption.

Evidence that the bottler complied with applicable statutes will be admissible to show that the bottler acted with ordinary, reasonable care, and is the only one of the listed arguments that would be helpful to the bottler. The bottler is being sued on a negligence theory; thus, the thief must prove that the bottler failed to exercise ordinary, reasonable care in bottling and distributing the water. Violation of a statute will establish a conclusive presumption of duty and breach of duty. However, compliance with an applicable statute does not necessarily establish due care, because due care may require more than is called for by the statute. Nevertheless, compliance with a statute is admissible as evidence that a defendant may have acted with due care. Thus, the bottler could use its compliance with the water bottling statutes as a means of establishing that it conformed with its duty to use ordinary, reasonable care. (A) is incorrect because a products liability action based on negligence uses the same causation analysis as a standard negligence case. Thus, a defendant’s liability is not cut off by a foreseeable intervening force that comes into motion after the defendant’s original negligent act. Consequently, an intermediary’s negligent failure to discover a defect is not a superseding cause, and the defendant whose original negligence created the defect will be held liable along with the intermediary. Hence, the retailer’s possibly negligent failure to inspect the water for purity will not relieve the bottler of liability for the consequences of its own negligence, if any. (C) will not be helpful to the bottler because this question allows for use of res ipsa loquitur. Under this doctrine, if a plaintiff shows that his injury is of a type that would not normally occur in the absence of negligence, and that such negligence is attributable to the defendant (e.g., by showing that the instrumentality causing the injury was in the exclusive control of the defendant), the trier of fact is permitted to infer the defendant’s negligence. Here, impurities would not normally get into the bottled water in the absence of negligence, and the fact that the container from which the thief drank was unopened allows the trier of fact to infer that the impurity entered the water due to negligence on the part of the bottler. Therefore, the thief is not required to introduce evidence as to how the impurity got into the water in order to prevail. (D) is incorrect because the bottler’s duty of due care in the context of products liability arises from having placed the water into the stream of commerce. Having done so, the bottler owes a duty to any foreseeable plaintiff, whether such person be an actual purchaser of the water or merely a user thereof. With the placing of the water into the stream of commerce, the thief is a foreseeable plaintiff as a drinker of the water, regardless of the fact that he obtained the water by means of theft.

91
Q

A commuter airplane crashed minutes after taking off. The day before the crash, the plane underwent a required periodic service inspection by an aviation maintenance company. While servicing the airplane, a mechanic reversed the placement of two key operational parts that control the directional movement of the plane. While the two parts appear identical in appearance they have completely opposite functions and are supposed to go into separate wings of the plane. Reverse installation of the parts on the wrong wing meant that when the pilot moved the controls to ascend, the plane would descend. Thus, the harder the pilot attempted to get the plane to ascend, the quicker the plane descended, ultimately leading to the crash. The manufacturer’s service manual for the plane mentioned that the parts could be reversely installed but there were no design features to protect against reverse installation. The industry standard required critical flight parts to contain design features, which preclude the possibility of reverse installation. Everyone on the plane perished in the crash.

In a failure to warn strict product liability suit for damages brought by the families of the crew and passengers, will the manufacturer of the airplane be successful in defending against liability?

A. Yes, because the negligence of the mechanics was the proximate cause of the accident.

B. Yes, because the parts were not unreasonably dangerous as designed.

C. No, because the industry standard demonstrates the manufacturer is negligent per se.

D. No, because the manufacturer failed to adequately warn of the possibility of reverse installation.

A

D. No, because the manufacturer failed to adequately warn of the possibility of reverse installation.

92
Q

A woman brought a negligence claim against her doctor for failing to warn her of the risks associated with an elective surgery. The surgery was a success, but there was a 50% chance that the woman would
be unable to have children, something she had mentioned wanting to the doctor on several occasions. During discovery, several depositions were scheduled, including the plaintiff’s, which was scheduled
first. At her deposition, the woman recounted conversations she had with the doctor, her memories of the procedure, and admitted that she was currently pregnant. Immediately following the deposition, and before the remaining depositions, the doctor moved for summary judgment on the negligence claim.

How should the court rule on the doctor’s motion?

(B) Motion denied, because the doctor breached his duty to the woman by failing to warn her of a material issue.

(D) Motion granted, because the surgery was elective.

(C) Motion granted, because the doctor’s failure to warn did not result in sufficient injury to the woman.

(A) Motion denied, because the doctor’s failure to warn caused the woman to consent to the surgery.

A

(C) Motion granted, because the doctor’s failure to warn did not result in sufficient injury to the woman.

93
Q

A woman was injured when she used an electric toothbrush, which, when on the highest setting, caught fire and burned the woman’s mouth. It appears that the defect was caused when the toothbrush was manufactured. The toothbrush was manufactured by a manufacturer, distributed to a retailer by a distributor, and sold in a store by the retailer. The retailer asked a dentist to put a display of the toothbrushes in his dental office to see if patients liked them. The woman bought the toothbrush from the dentist when she saw the display in his office after a routine cleaning. The woman now seeks to bring a strict products liability action against all possible parties for her injuries.

Against whom may the woman file suit?

A. Against the manufacturer, the distributor, the retailer, and the dentist.

B. Against the manufacturer, the distributor, and the retailer only.

C. Against the manufacturer only.

D. Against the manufacturer, the distributor, and the dentist only.

A

B. Against the manufacturer, the distributor, and the retailer only.

94
Q

A father and a son were playing catch in their back yard when their neighbor negligently felled a large oak tree. The tree crashed into the yard where the father and son were playing. The son was able to move out of the way, but the tree landed on the father and seriously injured him. In addition to suffering emotional distress at seeing his father injured, the son developed severe and chronic migraine headaches.

Under the majority rule, can the son successfully bring a direct claim for negligent infliction of emotional distress against the neighbor?

A. Yes, because he witnessed a physical harm occurring to a loved one.

B. Yes, because he was in the zone of danger and suffered a physical manifestation of his emotional distress.

C. No, because he was not truly at risk of being injured by the falling tree.

D. No, because he did not suffer shock resulting from the observance of the accident.

A

B. Yes, because he was in the zone of danger and suffered a physical manifestation of his emotional distress.

95
Q

A man attended a fund-raising party at the home of a friend. Because the friend was expecting a large crowd to attend, he purchased a number of chairs to use at the party. When the man sat in one of the chairs it collapsed, injuring him. The chair in question was manufactured by a chair company. Analysis of the broken chair revealed that one of the legs of the chair, which the company had purchased from a supplier, had been defectively manufactured such that it could not hold the weight of a normal adult. The company randomly tested the chair legs it purchased from the supplier for strength. Such a test would have revealed the defect in this particular leg.

If the man brings a negligence action against the company, which of the following most accurately states what he must show to make a prima facie case against the defendant?

A. The man must prove that the chair leg was dangerously defective and that it had not been tested by the company.

B. The man must prove that the chair leg was dangerously defective and that the defect would have been discovered but for the company’s failure to exercise due care.

C. The man must prove that the chair leg was dangerously defective, that it had been tested by the company, and that the tests failed to reveal the defect.

D. The man must prove that the chair leg was dangerously defective.

A

B. The man must prove that the chair leg was dangerously defective and that the defect would have been discovered but for the company’s failure to exercise due care.

Think Like a Lawyer

A plaintiff injured by a product can choose to sue in either strict products liability or negligence.

Step by Step Walkthrough

Step 1: Negligence requires: (1) duty; (2) breach of that duty; (3) causation, both actual and proximate; and (4) damages. Actual cause means the cause-in-fact of the injury. Legal cause, also known as proximate cause, means whether the defendant owed a duty of reasonable care to the plaintiff as one who could foreseeably be injured. A defendant owes a duty of reasonable care only to foreseeable plaintiffs, to act as the reasonable prudent person would under the same circumstances.

Step 2: First, note that the plaintiff is suing in negligence, and not in strict products liability. It is important to always have the fact pattern in context. Here, the chair leg caused the man’s injury. The call of the question is to summarize a prima facie negligence case against the chair company, which had bought all the chair legs from a supplier.

Step 3: Select the answer stating that the man must prove that the chair leg was dangerously defective and that the defect would have been discovered but for the company’s failure to exercise due care. The duty that the company owes to the man is to behave as the reasonable prudent chair company. A failure to inspect the chair legs for defects would be a breach of that duty, causing damages to the man, who is a foreseeable plaintiff, as someone using the chair.

Step 4: Discard the answer option saying that this specific chair leg had not been tested by the company. If the exercise of due care in the industry is found to be a random testing of chair legs, then meeting that standard of care will suffice for the company.

Step 5: Discard the answer option saying that there had to be a test that revealed it was defective, as this extreme situation may not be necessary to show a breach of the duty of care.

Step 6: Finally, discard the answer choice saying that the man need only prove the defect in the chair leg, since this would be enough for strict products liability, but not enough for negligence.

96
Q

The owner of a corner lot allowed a hedge on his property to become overgrown, obstructing the view of motorists at that corner. Two motorists were driving inattentively and each ran a stop sign at the intersection bordering the lot. Their cars collided in the intersection and one of the motorists was injured. She sued the owner of the lot. The jury determined that the lot owner was 10% at fault and each of the motorists was 45% at fault.

Will the injured motorist recover damages from the lot owner?

A. Yes, because she was not more than 50% at fault.

B. Yes, because it was foreseeable that motorists could be injured if the hedge was not cut back.

C. No, because the other motorist’s negligence was a superseding cause of her injuries.

D. No, because the lot owner’s fault was slight compared with the motorist’s fault.

A

B. Yes, because it was foreseeable that motorists could be injured if the hedge was not cut back.

The injured motorist may recover damages from the lot owner because the jury found that the lot owner should have foreseen that motorists could be injured if the hedge was not cut back. The lot owner owes the duty of an owner and occupier of land to those off the premises for unreasonably dangerous artificial conditions. In contrast to overgrown weeds, which are a natural condition for which no duty is owed absent a statute, a hedge is considered an artificial condition, analogous to a fence. Hence, by letting the hedge become so large that it created a foreseeable danger to motorists by obstructing their vision, the lot owner has breached his duty to the motorist. The other elements of the motorist’s negligence action (besides a duty and a breach of the duty) are actual and proximate cause, and damages. The motorist can establish actual cause by showing that, although she failed to notice the stop sign, she would have noticed another car traveling on a collision course with hers; i.e., but for the overgrown hedge, the motorist would have been able to avoid the accident. Proximate cause in an indirect case such as this can be established by showing that any intervening forces were foreseeable and not superseding. The other motorist’s negligent failure to stop may also have been caused in part by the overgrown hedge and is a foreseeable intervening force that does not break the chain of causation.

97
Q

A motorist was driving to a luncheon in a car that he knew did not have operating headlights. On the way there he was rear-ended by another driver who had been driving 20 m.p.h. over the speed limit posted on that stretch of road. He suffered personal injuries and his car was extensively damaged. The jurisdiction makes it a misdemeanor to drive a vehicle that does not have operating headlights.

If the motorist brings an action against the other driver and the above facts are established, will he prevail?

A. Yes, because the other driver violated the speeding statute, but the motorist’s damages will be reduced because of his violation of the headlight statute.

B. Yes, because the other driver violated the speeding statute, and the motorist’s damages will not be reduced despite his violation of the headlight statute.

C. No, because the motorist’s violation of the headlight statute constitutes negligence per se.

D. No, because the motorist has not established that driving 20 m.p.h. over the speed limit created an unreasonable risk of injury to others.

A

B. Yes, because the other driver violated the speeding statute, and the motorist’s damages will not be reduced despite his violation of the headlight statute.

98
Q

A company that is involved in oil drilling is interested in hiring a new engineer. The company interviews a few engineers until it meets a certain highly qualified engineer. During the interview, the company representative orally tells the engineer that they would be interested in hiring the engineer for at least 6 months. The engineer takes the company’s offerand the two parties sign an agreement. However, the agreement does not mention any duration term for the engineer’s employment.

Which of the following is correct?

A. The engineer was hired at-will, because that is the default rule.

B. The engineer was hired at-will, because he was not given any written assurances to the contrary.

C. The engineer was hired for at least 6 months, because that is what was told to him by the company representative.

D. The employment agreement is not valid, since it is missing an essential term.

A

C. The engineer was hired for at least 6 months, because that is what was told to him by the company representative.

Discussion of correct answer: In practically every jurisdiction, the employment-at-will rule is the default rule for the duration of an employment contract; absent agreement to the contrary, an employer may dismiss, and an employee may quit, at any time for any reason. However, in a majority of jurisdictions, oral or written assurances of job security made to an individual employee, as well as assurances contained in policy documents distributed to the workforce, may suffice to take the contract out of the default rule. Here, the company representative made oral assurances to the engineer that he would be hired for at least 6 months. Therefore the contract is not at-will and will be taken out of the default rule.

99
Q

A man asked the owner of an old pickup truck if he could borrow it to haul some firewood, and the owner agreed. Shortly after the man picked up the truck from its owner, the owner remembered that the truck’s steering had been damaged and was very sluggish, so he called the man’s wife and told her to tell the man about it. The man ran several errands and arrived home with the truck a couple of hours later. His wife forgot to mention the phone call. Later, while the man was driving the truck filled with firewood, the steering did not respond on a curve and the man hit a tree. If the truck’s steering had not been damaged, the man would not have hit the tree.

If the man asserts a claim for personal injuries against the owner of the truck, who is likely to prevail?

A. The man, because the owner failed to tell him that the steering on the pickup was damaged.

B. The man, because the owner was strictly liable for any damages caused by the defective steering in his truck.

C. The owner, because as a gratuitous bailor, his duty of care to the man was very slight.

D. The owner, because the wife’s failure to tell the man about the steering on the pickup was the cause in fact of the man’s injuries.

A

A. The man, because the owner failed to tell him that the steering on the pickup was damaged.

100
Q

A man was driving to work in the northbound lane of a highway at the same time that a mother was driving
her son to school in the southbound lane of the same highway. The son began to cry, which diverted
the mother’s attention from the road, causing her to drift slightly over the center line of the highway. The
man saw the mother’s vehicle cross the line and enter his lane. The man did not move over, nor did he
decrease his speed. As the mother drew closer, the man could have moved over to avoid the accident,
but did not, and the mother’s car hit the man’s car. The mother was injured and her car suffered damage.
The applicable jurisdiction has adopted a rule of contributory negligence. The mother subsequently
brought suit against the man for her injuries.

What is the most appropriate result at trial?

(A) The mother cannot recover damages, because she was negligent.

(D) The mother can recover damages, because she acted reasonably given the circumstances.

(B) The mother cannot recover damages, because she was negligent per se.

(C) The mother can recover damages, because the man could have slowed down or moved over.

A

(C) The mother can recover damages, because the man could have slowed down or moved over.

101
Q

In need of a new wristwatch, a shopper paid a visit to a watch store. The watch store occupied a two-story structure. Men’s wristwatches were displayed on the ground floor and women’s wrist watches were displayed on the second floor. Two escalators, one moving up and the other moving down, allowed customers to move quickly between the two floors. The company who installed the escalators was also responsible for their upkeep.

The shopper got on the escalator moving up to visit the second floor of the store. Once she was half way there, the shopper discovered that her foot had gotten trapped between the escalator stairs and the side rail and she could not remove it. The shopper screamed for help as she neared the top of the escalator. The owner of the watch store happened to be standing at the top of the escalator and heard the screams. The owner could have easily reached over and hit the escalator stop button, which would have allowed time to remove the shopper’s trapped foot without injury. However, the owner froze, unsure of what to do. As a result, when the shopper reached the top of the escalator her foot was badly mangled in the escalator mechanism.

If the shopper sues the owner for the injuries to her foot, what is the probable outcome?

A. The shopper will lose, because the owner did not create the peril.

B. The shopper will lose, because the owner was faced with an emergency situation.

C. The shopper will win, because she came to the store to purchase a wristwatch.

D. The shopper will win, unless responsibility was delegated to the escalator company.

A

C. The shopper will win, because she came to the store to purchase a wristwatch.

Discussion of correct answer: If the shopper came to the store to purchase a wristwatch she would be a business invitee in a place of public accommodation and would be owed a duty of reasonable care to aid or assist from the owner of the shop.

Although in general individuals do not have a duty to rescue those in need, an individual may have such a duty when there exists a special relationship with the potential victim. Such a relationship exists when the victim is a business invitee on the premises. In such a case, the business and its agents have a duty to intervene and use reasonable care to aid and assist the invitee. When the owner failed to intervene to aid the shopper, he breached his duty to her.

102
Q

While on vacation, two friends went scuba diving in the ocean. Each of them was equipped with a spear gun, which they planned to use to kill tuna. As they were swimming, both men saw what appeared to be a large fin entangled in a nearby coral reef. Simultaneously, they shot their spear guns at the projecting wing-like figure. Unbeknownst to either of the friends, it was not a fish but rather another scuba diver.
The victim was struck and slightly wounded by one of the spears. The victim asserts a claim for damages against the two friends.

At trial, how should the judge instruct the jury to rule?

(B) Against both defendants jointly, unless one of them proves that he did not shoot the spear that struck the victim.

(A) In favor of both defendants, if no evidence is presented showing who actually shot the spear that injured the victim.

(D) Against each defendant for one-half of the amount of damages, because they both shot their spear guns simultaneously.

(C) Against each defendant for one-half of the amount of damages, because they both acted independently and not jointly.

A

(B) Against both defendants jointly, unless one of them proves that he did not shoot the spear that struck the victim.

103
Q

Three friends decided to go bowling together. They made a friendly wager on the match, with the winner receiving a free beer from the other two. In the second frame of the first game, the first friend apparently got a strike when he knocked down all 10 pins. However, the second friend accused the first friend of fouling because his foot went over the line. The first friend denied fouling and wanted to mark down a strike on the scorecard. The second friend refused to give the first friend a strike and wrote in a zero on the scoring sheet. The first friend became enraged. The second friend then went to bowl his frame. As the second friend turned his back, the first friend approached from behind with a bowling ball in his hand. The first friend then raised the bowling ball, threatening to hit the back of the second friend’s head. The second friend, who had his back turned, did not see the first friend’s actions. The third friend saw what the first friend was about to do and could easily have warned the second friend; but the third friend remained silent. The first friend then struck the second friend on the back of the head with the bowling ball, causing a deep scalp wound.

If the second friend asserts a cause of action against the third friend, what is the likely outcome?

A. The second friend will prevail, because the third friend was aware of the danger.

B. The second friend will prevail, on account of the third friend’s omission to act.

C. The second friend will not prevail, because the first friend was responsible for his injury.

D. The second friend will not prevail, because the third friend was under no duty to warn the second friend of the danger.

A

D. The second friend will not prevail, because the third friend was under no duty to warn the second friend of the danger.

104
Q

A man purchases a widget from a retail outlet. The widget consists of three components. Each component was manufactured by a different manufacturer, and then all three components were assembled by a corporation which owns an assembly plant. While the man is using the widget, a defect causes the widget to malfunction, injuring the man. On further investigation, the malfunction turns out to have been caused by a defect in the third component. The man files a strict products liability suit for damages arising from his injuries.

Which of the following is correct?

A. The only liable party will be the manufacturer of component 3.

B. The corporation that assembled the components will not be liable.

C. The corporation that assembled the components and the manufacturer of component 3 will be liable.

D. The only liable parties will be the manufacturer of component 3 and the retailer.

A

C. The corporation that assembled the components and the manufacturer of component 3 will be liable.

105
Q

A pedestrian, who was walking along Chestnut Street at 10:20 p.m. on the night of December 3, urgently needed to find a restroom. Just ahead, the pedestrian noticed a private dinner club. As the pedestrian approached the club, he noticed a sign over the front door that read: “ADMITTANCE TO MEMBERS ONLY.” Although the pedestrian was not a member of the exclusive club, he entered the dimly lit club, found a door marked “Gentlemen,” and entered the restroom.

Which of the following would best describe the pedestrian’s legal status when he was in the restroom?

A. Trespasser.
B. Guest.
C. Licensee.
D. Invitee.

A

A. Trespasser.

106
Q

A partner at a law firm arranged for a car service to take him to work. Due to heavy traffic, the car service arrived 30 minutes late. As the car crossed over a bridge, an earthquake struck, causing the bridge to collapse and the vehicle to plummet three stories to the ground. The partner was seriously injured and was unable to return to work.

The partner was responsible for the majority of the firm’s litigation. In the partner’s absence, the firm had insufficient associates to cover the partner’s workload. As a result, the firm lost three potential clients. The firm and the partner have filed a suit against the car service.

Which of the following is the most accurate statement?

A. The car service will be liable to the partner, but not the firm.

B. The car service will be liable to the firm, but not the partner.

C. The car service will be liable to both the partner and the firm.

D. The car service will not be liable to either the partner or the firm.

A

D. The car service will not be liable to either the partner or the firm.

Discussion of correct answer: Both the firm and the partner would likely be asserting a negligence cause of action against the car service. Both would be able to show that a duty was owed, the car service breached that duty, and that there were damages as a result of that breach. However, both plaintiffs would still need to prove causation, both actual and proximate, to prove the elements of negligence. While actual cause is clear, proximate cause is not established on these facts. It is true that “but for” the car service’s tardiness, the car would not have been on the bridge when the earthquake struck. However, it is not foreseeable that being late to pick someone up would result in physical harm, nor was it foreseeable that being 30 minutes late would result in the car being in a more vulnerable location when an unexpected earthquake struck. Additionally, the firm cannot bring a cause of action for the loss of potential business due to the injuries sustained by an employee. Courts frown upon purely economic losses such as this, as businesses need to insure against such possibilities. Therefore, neither the partner nor the firm will recover against the car service for negligence. Note that there is no need for a discussion of superseding cause, as this is more germane to a situation where there is a subsequent injury sustained by the plaintiff and not directly caused by the defendant.

107
Q

A homeowner owned a backyard above-ground swimming pool. One day, she noticed that the pool filtration unit did not seem to be functioning properly. She determined that she needed a new filtration unit, so she went to the local pool supply store to purchase one. The homeowner looked for the model number for the filtration unit designed for her particular brand of pool, but noticed the store was out of that model. The homeowner saw another filtration unit that looked similar to the one she currently had. Although it was not manufactured specifically for her brand of pool, the homeowner decided it was close enough, so she purchased the filtration unit and took it home.

The homeowner installed the filtration unit, carefully following the manufacturer’s directions. When the installation was complete she activated the filtration unit. Because of a defect in the electrical system, the filtration unit shorted out, causing a spark that burned the homeowner’s hands.

In an action by the homeowner against the manufacturer, who will prevail?

A. The manufacturer, if it was unaware of and could not reasonably have discovered the defect in the electrical system.

B. The manufacturer, because the homeowner assumed the risk by installing the wrong model.

C. The homeowner, because she was injured by the defective electrical system.

D. The homeowner, because misuse is never a defense to a product liability action sounding in strict liability.

A

C. The homeowner, because she was injured by the defective electrical system.

108
Q

A caterer negligently used spoiled ingredients to prepare food for a wedding, and several guests who consumed the food fell violently ill with food poisoning. The guests called an ambulance to transport them to a hospital. On the way to the wedding to pick up the guests, the driver of the ambulance negligently ran a red light. The ambulance driver caused an accident with a motorist, injuring himself, another EMT worker who was riding in the ambulance, and the motorist.

The caterer will be liable for the injuries to whom?

A. The wedding guests only.

B. The wedding guests and the ambulance driver.

C. The wedding guests and the EMT worker.

D. The wedding guests, the ambulance driver, the EMT worker, and the motorist.

A

D. The wedding guests, the ambulance driver, the EMT worker, and the motorist.

Discussion of correct answer: The facts indicate that the caterer negligently used spoiled ingredients in the food she prepared, so she will be liable for the food poisoning suffered by the wedding guests without the need for further analysis. In addition, it is necessary to examine the rules relative to proximate causation dealing with rescuers. A negligent defendant owes an independent duty of care to a rescuer. Even where the rescue efforts are done negligently (but provided they are not wanton), the negligent defendant will be liable for both personal injury and property damage, whether the rescuer succeeds in injuring himself, the person rescued, or a stranger. Here, the facts indicate that the ambulance driver (the rescuer) was negligence in driving the ambulance. However, under the rescuer rule, the caterer will be liable for the injuries caused by the ambulance driver. As such, the caterer will also be liable for the injuries suffered by the ambulance driver, the EMT worker, and the motorist.

109
Q

Following a three-car accident, a woman sued a man for negligence, claiming that she suffered damages of $100,000. The driver of the third car was uninsured and the woman chose not to join him as a party. At trial, the jury found that the woman was 30% at fault, the man was 40% at fault, and the third driver was 30% at fault. The jury agreed that the woman’s damages were worth $100,000.

How much will the woman be able to recover from the man?

(C) $40,000, because that is the man’s percentage of fault.

(A) $100,000, because pure comparative negligence allows for recovery regardless of the plaintiff’s negligence, and the man can sue the third driver.

(D) Nothing, because her negligence was equal to the third driver’s negligence, precluding her recovery.

(B) $70,000, because a negligent plaintiff’s recovery should be reduced by her percentage of fault.

A

(B) $70,000, because a negligent plaintiff’s recovery should be reduced by her percentage of fault.

110
Q

A man purchased two tickets for a basketball game. The man then contacted his best friend, who agreed to accompany the man to the game. As they were driving to the game together, the man sped up to cross an intersection while the traffic signal was changing from yellow to red. As he reached the intersection, the man was traveling at twice the posted speed limit. Simultaneously, a car driven by a woman entered
the intersection despite having a red light and collided with the man’s car. The friend, who was not wearing a seatbelt at the time, suffered a broken arm and collarbone in the collision. This jurisdiction has adopted a modified comparative negligence statute. This jurisdiction has also adopted a statute requiring all occupants of motor vehicles to wear a safety belt, harness, or strap. The friend subsequently brought suit against the woman of the car that ran the red light to recover damages for his injury.

What is the most appropriate result?

(C) The friend’s recovery should be reduced, because he was negligent per see.

(B) The friend should recover full compensation, because he was not in control of either vehicle that caused the accident.

(A) The friend should recover full compensation, because he was not negligent.

(D) The friend should recover nothing, because he was negligent per se.

A

(C) The friend’s recovery should be reduced, because he was negligent per see.

111
Q

A rancher owned a 2,000-acre cattle ranch in a remote section of the state. For some time the rancher’s cattle had been seized upon by roaming packs of wolves. In order to protect his livestock from these attacks, the rancher erected a fence around his property. In addition, the rancher installed electrical wiring on the fence. The wiring transmitted an electric current, which would emit a shock upon contact with the fence. Although the voltage was calculated to frighten animals away from the fence, it was of such low intensity that it posed no risk of injury to a human being, even a small child. Consequently, the rancher did not post any notice of the electrical wiring device.

One day, a cowboy was riding his horse along the outskirts of the rancher’s property, when a gust of wind blew his hat onto the fence. The cowboy, who had undergone major heart surgery, wore a pacemaker. When the cowboy touched the top of the fence to retrieve his hat, the electric current short circuited his pacemaker, resulting in the cowboy’s death.

In a wrongful death action by the cowboy’s estate against the rancher, the installation of the electrical wiring device would most likely have what legal effect?

A. It will constitute the legal cause of the cowboy’s death, but not the cause-in-fact.

B. It will constitute the cause-in-fact of the cowboy’s death, but not the legal cause.

C. It will constitute both the legal cause and the cause-in-fact of the cowboy’s death.

D. It will constitute neither the legal cause nor the cause-in-fact of the cowboy’s death.

A

B. It will constitute the cause-in-fact of the cowboy’s death, but not the legal cause.

112
Q

A manufacturing company sold logging equipment for the general public. The company did not place warnings on its saws, axes, and other tools. A grocer, who was not a professional logger, bought one of the company’s axes from his local hardware store. While trying to cut some logs with his new axe, he grabbed the axe incorrectly, and hurt himself. The grocer sued the manufacturer.

Which of the following is correct?

A. The grocer prevails, because he was not a professional logger.

B. The grocer prevails, because the axe required a warning.

C. The company prevails, because it did not sell the axe to the grocer.

D. The company prevails, because the axe did not require a warning.

A

D. The company prevails, because the axe did not require a warning.

113
Q

A baseball fan purchased two tickets for a World Series baseball game. The fan contacted his best friend and invited him to go to the game. The friend, who was also a baseball fanatic, eagerly agreed. The fan told the friend that the game started at 7:00 p.m. and that he would pick him up at about 5:00 p.m. so they could get there early to watch batting practice. They were driving to the game together when the fan sped up to cross an intersection while the traffic signal was changing from amber to red. As he reached the intersection, the fan was traveling at 50 m.p.h. although the posted speed limit was 25 m.p.h. Simultaneously, a car entered the intersection on red and collided with the fan’s vehicle. The friend suffered a broken pelvis in the collision.

This jurisdiction has adopted the following “modified” comparative negligence statute: “A negligent plaintiff is entitled to obtain a recovery provided plaintiff’s negligence is not equal to or greater than that of the defendant’s; otherwise no recovery is permitted.”

Suppose the friend brings suit against the driver of the car that entered the intersection on the red light to recover damages for his injury. A jury returned a special verdict with the following findings: (1) The fan was 55 percent negligent in speeding; (2) The driver was 45 percent negligent in driving through the red light; and (3) The friend suffered $100,000 in damages.

As a result, the court should enter a judgment in favor of the friend for what amount?

A. $100,000.
B. $55,000.
C. $45,000.
D. Nothing, because the fan was more negligent than the driver.

A

A. $100,000.

On all standardized examinations (whether it be the MBE, SAT, LSAT, or GRE), the examiner’s main goal is to hide the correct answer. The examiners try to accomplish this by employing “distracters” and “red herrings.” Here, the comparative negligence statute is the distracter. A comparative negligence statute only applies when there is a negligent plaintiff and one or more negligent defendants. Here, there were two negligent parties (the fan and the driver), but neither of them is the plaintiff. The plaintiff (the friend) was just a passenger who was riding in one of the cars and did nothing negligent. The comparative negligence statute, therefore, is irrelevant. The two negligent parties (the fan and the driver) are joint tortfeasors who can be held jointly and severally liable for the full amount of the plaintiff’s damages. This answer choice is correct because the friend may recover 100% of the damages from the driver, who could then seek contribution from the fan.

114
Q

A wife and her husband were having dinner at a restaurant when the wife excused herself to go to the bathroom. The restaurant was owned and operated by a chef. As the wife was walking past a table where another customer was seated, she slipped and fell on an egg roll that had been lying on the floor for quite some time, although the chef was unaware that it had fallen onto the floor. When she fell, her head struck a serving tray that was located in the aisle. The fall caused the wife to suffer a severe concussion. The customer knew that the egg roll was on the floor, and although he could have done so, he did not warn the wife.

If the wife asserts a claim against the chef for the injuries she suffered from the fall, what is the most likely outcome?

A. The wife will recover, because the egg roll on the floor constituted an unsafe condition of the premises.

B. The wife will recover, because the egg roll was on the floor for a substantial period of time before the accident.

c. The wife will not recover, because the chef did not know that the egg roll was on the floor.

D. The wife will not recover, because the customer could have prevented the injury by warning the wife of the presences of the egg roll.

A

B. The wife will recover, because the egg roll was on the floor for a substantial period of time before the accident.

115
Q

A woman was walking her dog one day around her neighborhood. As she passed a neighbor’s house, the neighbor’s dog charged out from the side of the property and attacked the woman. A passerby helped get the neighbor’s dog off the woman, though she was seriously injured from the attack. The
woman later brought a claim against the neighbor for her medical bills. At trial, the neighbor stated that he was diligent in keeping his dog in the backyard, and therefore, someone must have let it out, which was not his fault. The neighbor then filed a motion for judgment as a matter of law.

How should the court rule on the neighbor’s motion?

(B) Denied, because the keeper of an animal is strictly liable for injuries that it causes.

(D) Granted, because the woman cannot prove that someone else did not let the neighbor’s dog out of the backyard.

(C) Granted, because the neighbor testified that he was diligent in keeping his dog in the backyard.

(A) Denied, because the dog was within the control of the neighbor at the time of the injury.

A

(A) Denied, because the dog was within the control of the neighbor at the time of the injury.

116
Q

A manufacturer of component parts supplies wheels to a large scooter producer. One of the wheels ends up being defective, but is still placed on a scooter, because the producer is not aware that the wheel is defective. The scooter is later sold to a mother who buys it for her daughter. Three weeks after the purchase of the scooter the defective wheel breaks when the daughter is riding her scooter at a local park. As a result, the girl crashes into an elderly woman who suffers serious injuries. The elderly woman files a strict product liability action against the scooter producer.

Which of the following is true?

A. The elderly woman will prevail, because the scooter producer is in the chain of distribution.

B. The elderly woman will prevail, because of the consumer expectation test standard.

C. The scooter producer will prevail, because the defect is the fault of the manufacturer of component parts.

D. The scooter producer will prevail, because the elderly woman was not using the defective product.

A

A. The elderly woman will prevail, because the scooter producer is in the chain of distribution.

117
Q

A vacation resort sold time-share vacations on their property. To entice prospective buyers to visit their property the resort would offer free weekend stays to anyone interested in possibly purchasing a time-share. A guest decided to take advantage of the free weekend offer. One of the terms of the free stay was that the guest had to go on a guided tour of the property to see what the resort had to offer. The guided tours were conducted by having a sales representative for the resort drive the prospective buyer around the property on a golf cart.

When the guest showed up at the appointed time for his guided tour he was informed that there was only one golf cart left that could be used for the tour, but the seat belt on the passenger’s side did not always work properly. The guest was reminded that a state law prohibited anyone from riding as a passenger in any four-wheel motorized vehicle without a working seat belt and that the guest would be in violation of that law if the one golf cart available was used. The guest was informed that another tour was scheduled to end in 15 minutes, and that if he would not mind waiting, that golf cart had working seat belts. The guest responded that he was not going to wait and that he would either go on the tour that second or not go at all. The sales representative reluctantly agreed to take the guest on the tour. Twenty minutes later, while the guest was in the middle of the tour, another sales representative for the resort, who had just finished a guided tour, stopped the sale representative escorting the guest around the property and offered his golf cart for use in the guest’s tour. The exchange was made, the guest fastened his seat belt, andthe tour continued. Five minutes later the guest was injured when the golf cart in which he was riding was hit by a service truck driven by a resort employee.

The guest filed suit against the resort for his injuries. The jurisdiction in question has adopted pure comparative negligence approach. The resort is defending on the grounds that the guest’s violation of the seat belt statute constituted negligence per se, arguing that compliance with the statute would have delayed the guest by 15 minutes, so that he would not have been present when the service truck came along that ultimately hit him.

Which of the following best supports a rejection of this defense by the trial court?

A. The guest’s negligence, if any, is no defense because the resort is strictly liable.

B. The seat belt statute was not intended to protect passengers by controlling the timing of their visits.

C. The guest was wearing his seat belt when the truck collision occurred.

D. The driver of the service truck had the last clear chance to avoid injuring the guest.

A

B. The seat belt statute was not intended to protect passengers by controlling the timing of their visits.

118
Q

A mining company closed down operations at an isolated mine it owned and informed the electric company that electricity in the power poles that led to the mine should be cut off. However, the electric company, following its standard policy, left the power running in the line to deter thieves from stealing valuable transformers and cables. The mining company was unaware that the power was left on.

A hitchhiker who was passing by the entrance to the mine saw that it was closed, so he went onto the property and climbed up a power pole to steal a transformer. He received an electric shock and fell from the pole, suffering serious injuries.

If the hitchhiker sues the mining company, which of the following is the mining company’s strongest defense?

A. The hitchhiker was a trespasser.

B. The hitchhiker was a thief.

C. The mining company asked the utility company to turn off the power.

D. The mining company was unaware that the utility company had not turned off the power.

A

A. The hitchhiker was a trespasser.

119
Q

A man decided to take a vacation after a long month of work. He packed his luggage into a car and went on a cross country trip to a distant state. On the second day of the trip, the man was passing a dense forest when his car flipped over because of a defective tire. Ten minutes later, a driver was passing the site of the accident and saw the man unconscious in his overturned car. The driver got out of his vehicle and pulled the man from the wreckage. However, as the driver was pulling out the man, he accidentally got cut by a large of piece of metal that was sticking out of the car. The driver files a strict product liability action against the manufacturer of the tire for his injury. It is shown that the tire was manufactured in a form other than the one the manufacturer intended.

Should the driver prevail in his lawsuit?

A. Yes, because of the danger-utility test approach.

B. Yes, because he was a rescuer.

C. No, because the driver is not a proper plaintiff.

D. No, because the driver assumed the risk when he decided to pull the man out of the wreckage.

A

B. Yes, because he was a rescuer.

120
Q

A father took his teenage son to a department store to buy school clothing and a laptop computer for the new school year. The son had a psychiatric disorder which included paranoid delusions. While they were looking at a laptop computer, the son began to cry uncontrollably and scream that his father was trying to kill him. A store employee, a security guard, approached the father and told him to leave the son alone. The father explained to the employee he was the son’s parent and his son had a medical condition. When the father attempted to prevent the son from leaving, the security guard grabbed the father’s arm and knocked him to the ground. The father suffered bruises on his face and a broken wrist. The father filed a suit for battery naming both the employee and the store as defendants. The store has moved to dismiss the complaint, arguing that it cannot be held liable for its employee’s conduct.

Will the court grant the store’s motion to dismiss?

A. Yes, because the store cannot be held liable for the intentional torts of its employee.

B. Yes, because the store cannot be held liable for the negligence of an independent contractor.

C. No, because the store is liable for the negligence and intentional torts of any employee.

D. No, because the store could be liable if the employee was acting to further a purpose of the store.

A

D. No, because the store could be liable if the employee was acting to further the purpose of the store.

Discussion of correct answer: Under a theory of respondeat superior, an employer can be held vicariously liable for the negligence or strict liability of its employee when the employee’s actions are the cause of the plaintiff’s injuries. The employer will be held liable for the negligence of an employee only when the employee is acting within the scope of his employment. Generally, an employer will not be subject to liability for the intentional torts of an employee. However, if the employee commits an intentional tort that is in furtherance of the purposes of the employer and is done within the scope of employment, the employer could be liable under a respondeat superior theory. Here, the facts indicate the security guard did use force that injured the father, but they also indicate the force was used to protect the son and was done in furtherance of the employer’s interest to have a safe store.

Think Like a Lawyer

If your client is an employer, the client can be held liable for intentional torts of his employees, if those torts are committed to further your client’s business purposes.

Step by Step Walkthrough

Step 1: An employer is liable for injuries caused by the negligence or strict liability of an employee if the tortious act occurred within the scope of the employment. Intentional torts committed by an employee are generally not given respondeat superior effect, even if committed during working hours. However, if an employee uses force, even misguidedly, wholly or partly to further the employer’s purpose, such use of force may fall within the scope of employment, resulting in vicarious liability for the employer.

Step 2: Here, the call of the question is whether the court will grant the store’s motion to dismiss. The store is liable for the actions of its employee, which appear to be clearly within the scope of employment. The employee was trying to take care of a situation in the store that might have involved a man harassing a boy (when, in fact, they were father and son). This was part of the job duties of the employee.

Step 3: Consequently, under the doctrine of respondeat superior, the store is liable for the employee’s actions. The motion to dismiss will be denied. Select the answer stating this, because the store could be liable if the employee was acting to further a purpose of the store.

Step 4: Discard the answer choice saying that the store cannot be held liable for the intentional torts of its employee, because the store will be liable if the intentional tort was performed in furtherance of the store’s business.

Step 5: Discard the answer choice regarding an independent contractor, because the security guard was an employee, not an independent contractor.

Step 6: Finally, discard the answer choice saying that the store is liable for the negligence and intentional torts of any employee, because this is an overly broad, and thus incorrect, statement of the law. The store is only liable for battery if the employee committed the battery to further the store’s interests.

121
Q

A father regularly observed his 13-year-old son taunting the eight-year-old neighbor. On several occasions, the father overheard his son threatening to “beat up” the neighbor, but the father never took action to intervene in his son’s behavior, figuring “boys will be boys.” Recently, the son hit the neighbor with a baseball bat, causing serious injury to the neighbor.

Will the father likely be liable for the son’s actions?

A. Yes, because the father had the ability to exercise control over his child and failed to do so.

B. Yes, because a parent always has a duty to exercise reasonable care to control his minor child.

C. No, because a parent never has a duty to exercise reasonable care to control his minor child.

D. No, because the father did not know that the son was going to attack the neighbor.

A

A. Yes, because the father had the ability to exercise control over his child and failed to do so.

122
Q

A company operated a small amusement park on property it owned near a residential neighborhood. On a day when the park was closed, a 10-year-old girl snuck into the park with some friends by climbing over a chain link fence. While climbing on one of the carnival rides, the girl slipped and cut her leg on an exposed gear assembly, sustaining serious injuries.

Through her guardian ad litem, the girl brought suit against the company to recover damages for her injuries. At trial, she presented evidence of the accident and her injuries. In defense, the company established that the girl read and understood the “No Trespassing” signs that were attached to the fence. The company also established that it had not had any previous reports of children sneaking into the park when it was closed. Before submission of the case to the jury, the company moved for summary judgment.

Is the court likely to grant the company’s motion?

A. Yes, because the girl was a trespasser who the company had no reason to anticipate would be on the property.

B. Yes, because the girl knew she was trespassing and was old enough to recognize the danger.

C. No, because the jury could find that the company should have foreseen that children would sneak into the park.

D. No, because the appeal of the carnival rides attracted the girl into the park.

A

C. No, because the jury could find that the company should have foreseen that children would sneak into the park.

123
Q

A woman owned a used bookstore. One day, a customer came into the store drinking a large soda. He
carelessly dropped it on the floor, and the bookstore owner failed to clean it up. About four hours later, a second customer came into the bookstore, hoping to find a first edition of a book. The second customer,
trying to break in a new pair of five-inch heels that were a size too small, could barely walk. As she was teetering up and down the aisles browsing the selection, she slipped on the wet floor and broke her ankle. The second customer sued the bookstore owner, and after the trial, the jury found the second
customer 20% at fault, the bookstore owner 40% at fault, and the first customer 40% at fault. This jurisdiction has a modified comparative negligence statute.

Which of the following is most accurate?

(A) The second customer can recover, because she was not more negligent than the owner or the first customer.

(B) The second customer can recover, because she was not more negligent than the owner and the first customer.

(C) The second customer can recover as long as she did not have the last clear chance to avoid the injury.

(D) The second customer cannot recover, because she has contributed to her own negligence.

A

(B) The second customer can recover, because she was not more negligent than the owner and the first customer.

124
Q

A nurse was employed to care for patients at a nursing home. While doing her rounds one day, the nurse observed a man rummaging through the purse of one of the female patients. The nurse immediately jumped on the man and held him down until help arrived. The man then sued the nursing home for battery. The nursing home moved for summary judgment.

The court should:

A. Grant the motion.

B. Grant the motion because employers are not vicariously liable for the intentional torts of employees.

C. Deny the motion because the nurse acted outside the scope of employment.

D. Deny the motion because the nurse was negligent in jumping on the man.

A

A. Grant the motion.

125
Q

Even though he knew that the city had an ordinance forbidding the riding of skateboards on city sidewalks, a man was riding his skateboard on a sidewalk in the city when he collided with a woman as she was leaving a jewelry store. The woman fell to the ground in great pain. A passerby telephoned 911 for help and the paramedics quickly arrived. The paramedics determined that the woman had broken her leg. They put a splint on the leg and summoned an ambulance to transport the woman to the hospital. When the ambulance arrived, the woman was placed in the back. The paramedics told the ambulance driver that there was no need to rush to the hospital since she was resting comfortably. However, the ambulance driver had a bet with a fellow driver that he could handle more calls in one day. The ambulance driver knew he had to rush to the hospital to drop the woman off so he could pursue another call. He therefore drove very quickly to the hospital. The ambulance driver lost control at a sharp curve because he was traveling too fast. The ambulance overturned, injuring the woman’s back.

If the woman sues the man for her back injury, what will be the impact of the ambulance driver’s actions?

A. None, the man will be liable for all her injuries because rescuers are always foreseeable.

B. None, the man will be liable for all her injuries because the negligence of a rescuer is foreseeable.

C. The man will not be liable for the woman’s back injury because the ambulance driver’s actions were reckless.

D. The man will not be liable for any of the woman’s injuries because the ambulance driver’s reckless conduct was a superseding intervening force.

A

C. The man will not be liable for the woman’s back injury because the ambulance driver’s actions were reckless

Discussion of correct answer: If a rescue effort is foolish or reckless, it is deemed to be the sole legal cause of any additional injury resulting if the injuries are divisible. The woman suffered a broken leg in the original collision. Her back was injured in the rescue. These are apportionable injuries. Due to the fact that the ambulance driver was reckless in driving the ambulance very quickly, he will be solely responsible for the woman’s back injury

126
Q

A homeowner bought “20-pound test” fishing line for hanging potted plants on his porch. “20-pound test” in the fishing industry means that fishing line will not break under an initial stress of up to 20 pounds when a hooked fish tugs against the line, but not that it will support a constant 20-pound weight. Most sportfishers are aware of this technical meaning, but most laypersons are not, and the manufacturer put no warnings or explanations on the package in which the line was sold. The homeowner hung a 15-pound basket from his front porch, directly above an old-fashioned porch swing. A friend visiting the homeowner was sitting on the swing when the line holding the basket broke, causing the plant to fall and strike the guest on the head.

In a jurisdiction following the traditional rules for landowners and possessors of land, will the guest prevail against the homeowner in a suit to recover damages for her injuries?

A. Yes, because she was a social guest.

B. Yes, because the homeowner was negligent in hanging the plant.

C. No, because the homeowner could not be expected to know the technical meaning of “20-pound test.”

D. No, because she was not a foreseeable plaintiff.

A

C. No, because the homeowner could not be expected to know the technical meaning of “20-pound test.”

The guest will not prevail in a suit against the homeowner. As a social guest of the homeowner’s, the guest is deemed to be a licensee; i.e., one who enters onto land with the owner’s permission for her own purpose or business rather than for the owner’s benefit. In a jurisdiction following the traditional rules for landowners and possessors of land, the owner owes a licensee the duty to warn of or make safe a dangerous condition known to the owner that creates an unreasonable risk of harm to the licensee and that the licensee is unlikely to discover. The owner has no duty to a licensee to inspect for defects nor to repair known defects. The homeowner, as a person who was not involved with fishing, had no reason to suspect that a fishing line that was “20-pound test” could not support the constant weight of a 15-pound basket. Thus, the homeowner did not know of the dangerous condition present in the form of the basket overhanging his porch. Because the homeowner was unaware of the danger, he was under no duty to warn the guest, a licensee, of the dangerous condition. Having violated no duty owed to the guest, the homeowner will not be held liable for her injuries.

127
Q

A construction company was replacing large portions of a sidewalk in the downtown section of a city. The construction crew started tearing out the old sidewalk, and left for the day without putting up warning signs or barriers around the work area. A few hours later, a jogger came around the corner and fell in a hole at the spot where the construction crew had torn up the sidewalk. As the jogger was attempting to get out of the hole, she began screaming for help. Her screams attracted
the attention of a passerby, who immediately hurried to assist the jogger. As the passerby was leaning over the edge of the work area, he too fell into the hole. Both the jogger and the passerby suffered serious bodily injuries before they were pulled out of the hole by police. This jurisdiction has not adopted a
“Good Samaritan” statute.

In a negligence action by the passerby and the jogger to recover for their personal injuries, what is the most likely outcome?

(A) The jogger will prevail but not the passerby, because there was no “Good Samaritan” statute requiring him to come to the jogger’s aid.

(D) Neither will recover, because the jogger was running at night and the passerby had no relationship to her.

(B) The passerby will prevail because danger invites rescue, but the jogger will not recover because she created her own peril.

(C) Both will recover, because the jogger was injured by the negligence of the construction company and the passerby acted reasonably given the situation.

A

(C) Both will recover, because the jogger was injured by the negligence of the construction company and the passerby acted reasonably given the situation.

128
Q

Two nine-year-old boys, Pete and Victor, were sitting across from each other in their classroom at school. Pete kicked Victor on his shin so lightly that Victor did not immediately feel it. However, minutes later, Victor was crying out in pain due to the injury. Over the next week, the wound swelled and surgery was performed on Victor’s shin. The surgeons found that Victor’s shin bone was damaged from an earlier sledding accident and the kick that Pete delivered exacerbated the injury, causing bone degradation and irreparable harm to the leg. Due to these events, Victor would never be able to use the leg again. Victor’s parents, on his behalf, subsequently brought suit against Pete for the damages that Victor incurred.

What is the most likely outcome?

A Victor should recover for all of his injuries, because he was not at fault.
B Victor should recover for all of his injuries, because the eggshell plaintiff rule provides that a defendant takes their victim as they find them.
C Victor should only recover for those injuries sustained by the initial kick, but not for those attributable to the earlier sledding accident.
D Victor should not recover, because a nine-year-old boy cannot be liable for either negligence or intentional torts.

A

B Victor should recover for all of his injuries, because the eggshell plaintiff rule provides that a defendant takes their victim as they find them.

129
Q

A bicyclist was riding his bicycle in the street when a negligently driven car struck the bike, knocking the bicyclist off the bike and breaking his right ankle. The driver of the car immediately stopped and went to his assistance. She got him to his feet and was slowly moving him toward the curb when a negligently driven taxicab struck him in the left leg. The bicyclist required surgery on both his right ankle and his left leg.

If the bicyclist sues the driver and the cabbie, which of the following best states his right to recover?

A) He can recover from either the driver or the cabbie for all of his injuries because the driver and the cabbie are jointly and severally liable.

B) He can recover from the driver only for the injury to his right ankle and recover from the cabbie only for the injury to his left leg.

C) He can recover from either the driver or the cabbie for the injury to his left leg and recover from the driver only for the injury to his right ankle.

D) He cannot recover against the driver for the injury to his left leg unless the jury determines that the driver acted negligently when she came to his aid.

A

C) He can recover from either the driver or the cabbie for the injury to his left leg and recover from the driver only for the injury to his right ankle.

130
Q

A motorist purchased a new sport utility vehicle from his local dealer. Standard equipment on the vehicle included a set of top-of the-line tires from a premium tire company. However, the motorist was able to save $400 on the purchase price by allowing the dealer to substitute a lower priced tire manufactured by a discount tire manufacturer. Unbeknownst to the motorist and the dealer, the tire manufacturer had negligently designed the tires, with the result that a tire would occasionally blow out when the car was traveling at a high rate of speed in hot weather. On an exceptionally hot day, the motorist was traveling 80 m.p.h. in a 55 m.p.h. zone. A tire exploded, resulting in damage to the vehicle and injury to the motorist.

If the motorist sues the dealer on a theory of strict liability, is he likely to prevail?

A) Yes, because the tire was in a dangerously defective condition when the motorist purchased the car.

B) Yes, because the dealer is responsible for the negligence of the tire manufacturer, because the dealer used its tires.

C) No, because the motorist assumed the risk when he substituted the discount tires in exchange for $400.

D) No, because the motorist was misusing the tire when he was traveling at 80 m.p.h.

A

A) Yes, because the tire was in a dangerously defective condition when the motorist purchased the car.